POPULARITY
În cadrul ediției de pe 20 mai a emisiunii Știința360 de pe Radio România Cultural, Dr. Marius Geantă, Președintele Centrului pentru Inovație în Medicină, a comentat ultimele noutăți din domeniul sănătății. Utilizarea dispozitivelor medicale pe bază de inteligență artificială ar putea avea un rol semnificativ în screening-ul cardiovascular al femeilor care urmează să devină mame sau care își doresc să facă acest pas în viitor. Oamenii de știință de la Mayo Clinic, SUA, au analizat performanțele unui instrument EKG-AI și a unui stetoscop inteligent în detectarea cardiomiopatiei în rândul femeilor aflate la vârstă fertilă. Publicat în Annals of Family Medicine, studiul arată cum cele două dispozitive digitale semnalează cu acuratețe prezența fracției de ejecție a ventriculului stâng (LVEF) sub 50%, indicator cunoscut în cazul unui mușchi cardiac slăbit.În fiecare an, aproximativ 300.000 de femei își pierd viața ca urmare a complicațiilor apărute în timpul sarcinii sau al nașterii. O parte din aceste complicații pot fi prevenite, evitate sau gestionate cu mai multă siguranță, cu cât viitoarea mamă și medicii care o îngrijesc cunosc mai multe detalii despre starea sa de sănătate. Ziua Mondială a Sănătății din acest an a adus în prim-plan sănătatea mamelor și a nou-născuților, subliniind importanța măsurilor preventive luate din timp. Screening-ul cardiovascular înainte și în timpul sarcinii, realizat prin mijloace digitale precum dispozitivele bazate pe AI, nu numai că este rapid și cost-eficient, dar poate depista timpuriu femeile aflate la risc și poate informa viitorul planul de îngrijire al acesteia.Mai multe detalii despre subiectele discutate - ▶ Dispozitivele digitale de tipul AI-EKG, utile în screeningul cardiovascular al femeilor care doresc să devină mame▶ CORDELIA, cel mai amplu studiu asupra sănătății cardiovasculare în Europa de Sud. Datele genomice, sociale și de mediu vor contribui la strategii de prevenție personalizată▶ Un program de screening pentru cancerul colorectal desfășurat timp de 20 ani a demonstrat reducerea cu 50% a mortalității▶ Prima terapie CRISPR personalizată salvează viața unui bebeluș cu o boală genetică rară.Ascultă emisiunea pe Radio România Cultural.
Hosts Mitsuaki Sawano, MD, co-host Shun Kohsaka, MD, and Nobuhiro Ikemura, MD, welcome Tomohiko Taniguchi, MD, from Kobe City Medical Center General Hospital, to discuss findings from the CURRENT AS Registry-2 on low-gradient severe aortic stenosis (AS). Based on data from over 3,300 patients across 21 Japanese centers, the study categorized AS into four hemodynamic subtypes, revealing that low-gradient AS with reduced LVEF had the poorest prognosis, paradoxical LFLG AS was associated with high rates of atrial fibrillation and advanced cardiac damage despite low valve calcification, and normal-flow LG AS frequently exceeded guideline-based calcification thresholds despite being labeled as moderate. Dr. Taniguchi emphasizes the need to revise conventional illustrations of AS subtypes by incorporating cardiac damage and frailty and underscores the importance of proper statistical interpretation in subgroup analyses.
Join CardioNerds Heart Failure Section Chair Dr. Jenna Skowronski, episode lead Dr. Merna Hussein, and expert faculty Dr. Milton Packer as they discuss the SUMMIT trial. The SUMMIT trial randomized 731 patients with HFpEF with LVEF ≥ 50% and obesity with BMI ≥ 30 kg/m2 to receive tirzepatide or placebo for at least 52 weeks. The two co-primary endpoints were a composite of time to cardiovascular death or a worsening heart failure event and quality of life measured by the Kansas City Cardiomyopathy Questionnaire clinical summary score (KCCQ-CSS). Treatment with tirzepatide led to a lower risk of the composite of cardiovascular death or worsening heart failure as well as improved quality of life. This episode was planned in collaboration with the American College of Cardiology Section of the Prevention of Cardiovascular Disease with mentorship from Section Chair Dr. Eugenia Gianos. CardioNerds Journal Club PageCardioNerds Episode PageCardioNerds AcademyCardionerds Healy Honor Roll CardioNerds Journal ClubSubscribe to The Heartbeat Newsletter!Check out CardioNerds SWAG!Become a CardioNerds Patron! References - The SUMMIT Trial Packer, M., Zile, M. R., Kramer, C. M., Baum, S. J., Litwin, S. E., Menon, V., Ge, J., Weerakkody, G. J., Ou, Y., Bunck, M. C., Hurt, K. C., Murakami, M., Borlaug, B. A., & SUMMIT Trial Study Group. (2024). Tirzepatide for Heart Failure with Preserved Ejection Fraction and Obesity. The New England Journal of Medicine. https://doi.org/10.1056/NEJMoa2410027
William H. Sauer, MD, FHRS, CCDS, Brigham and Women's Hospital is joined by Edoardo Bressi, MD Queen Elizabeth Hospital - University Hospitals Birmingham, and Jordana Kron, MD Virginia Commonwealth University to discuss the current guidelines that present varying classes of recommendations for implantable cardioverter-defibrillator (ICD) utilization in patients with cardiac sarcoidosis (CS) and left ventricular ejection fraction (LVEF)
The following question refers to Sections 7.3.3 and 7.3.6 of the 2022 ACC/AHA/HFSA Guideline for the Management of Heart Failure.The question is asked by Palisades Medical Center medicine resident & CardioNerds Academy Fellow Dr. Maryam Barkhordarian, answered first by UTSW AHFT Cardiologist & CardioNerds FIT Ambassador Dr. Natalie Tapaskar, and then by expert faculty Dr. Robert Mentz.Dr. Mentz is associate professor of medicine and section chief for Heart Failure at Duke University, a clinical researcher at the Duke Clinical Research Institute, and editor-in-chief of the Journal of Cardiac Failure. Dr. Mentz has been a mentor for the CardioNerds Clinical Trials Network as lead principal investigator for PARAGLIDE-HF and is a series mentor for this very Decipher the Guidelines Series. For these reasons and many more, he was awarded the Master CardioNerd Award during ACC22.The Decipher the Guidelines: 2022 AHA / ACC / HFSA Guideline for The Management of Heart Failure series was developed by the CardioNerds and created in collaboration with the American Heart Association and the Heart Failure Society of America. It was created by 30 trainees spanning college through advanced fellowship under the leadership of CardioNerds Cofounders Dr. Amit Goyal and Dr. Dan Ambinder, with mentorship from Dr. Anu Lala, Dr. Robert Mentz, and Dr. Nancy Sweitzer. We thank Dr. Judy Bezanson and Dr. Elliott Antman for tremendous guidance.Enjoy this Circulation 2022 Paths to Discovery article to learn about the CardioNerds story, mission, and values. American Heart Association's Scientific Sessions 2024As heard in this episode, the American Heart Association's Scientific Sessions 2024 is coming up November 16-18 in Chicago, Illinois at McCormick Place Convention Center. Come a day early for Pre-Sessions Symposia, Early Career content, QCOR programming and the International Symposium on November 15. It's a special year you won't want to miss for the premier event for advancements in cardiovascular science and medicine as AHA celebrates its 100th birthday. Registration is now open, secure your spot here!When registering, use code NERDS and if you're among the first 20 to sign up, you'll receive a free 1-year AHA Professional Membership! Question #39 Ms. Kay Lotsa is a 48-year-old woman with a history of CKD stage 2 (baseline creatinine ~1.2 mg/dL) & type 2 diabetes mellitus. She has recently noticed progressively reduced exercise tolerance, leg swelling, and trouble lying flat. This prompted a hospital admission with a new diagnosis of decompensated heart failure. A transthoracic echocardiogram reveals LVEF of 35%. Ms. Lotsa is diuresed to euvolemia, and she is started on carvedilol 25mg BID, sacubitril/valsartan 49-51mg BID, and empagliflozin 10mg daily, which she tolerates well. Her eGFR is at her baseline of 55 mL/min/1.73 m2 and serum potassium concentration is 3.9 mEq/L. Your team is anticipating she will be discharged home in the next one to two days and wants to start spironolactone. Which of the following is most important regarding her treatment with mineralocorticoid antagonists?ASpironolactone is contraindicated based on her level of renal impairment and should not be startedBSerum potassium levels and kidney function should be assessed within 1-2 weeks of starting spironolactoneCEplerenone confers a higher risk of gynecomastia than does spironolactoneDThe patient will likely not benefit from initiation of spironolactone if her cardiomyopathy is ischemic in origin Answer #39 ExplanationThe correct answer is B – after starting a mineralocorticoid receptor antagonist (MRA), it is important to closely monitor renal function and serum potassium levels.MRA (also known as aldosterone antagonists or anti-mineralocorticoids) show consistent improvements in all-cause mortality, HF hospitalizations, and SCD across a wide range of patients with HFrEF.
The following question refers to Sections 7.4 and 7.5 of the 2022 AHA/ACC/HFSA Guideline for the Management of Heart Failure.The question is asked by the Director of the CardioNerds Internship Dr. Akiva Rosenzveig, answered first by Vanderbilt AHFT cardiology fellow Dr. Jenna Skowronski, and then by expert faculty Dr. Randall Starling.Dr. Starling is Professor of Medicine and an advanced heart failure and transplant cardiologist at the Cleveland Clinic where he was formerly the Section Head of Heart Failure, Vice Chairman of Cardiovascular Medicine, and member of the Cleveland Clinic Board of Governors. Dr. Starling is also Past President of the Heart Failure Society of America in 2018-2019. Dr. Staring was among the earliest CardioNerds faculty guests and has since been a valuable source of mentorship and inspiration. Dr. Starling's sponsorship and support was instrumental in the origins of the CardioNerds Clinical Trials Program.The Decipher the Guidelines: 2022 AHA / ACC / HFSA Guideline for The Management of Heart Failure series was developed by the CardioNerds and created in collaboration with the American Heart Association and the Heart Failure Society of America. It was created by 30 trainees spanning college through advanced fellowship under the leadership of CardioNerds Cofounders Dr. Amit Goyal and Dr. Dan Ambinder, with mentorship from Dr. Anu Lala, Dr. Robert Mentz, and Dr. Nancy Sweitzer. We thank Dr. Judy Bezanson and Dr. Elliott Antman for tremendous guidance.Enjoy this Circulation 2022 Paths to Discovery article to learn about the CardioNerds story, mission, and values. American Heart Association's Scientific Sessions 2024As heard in this episode, the American Heart Association's Scientific Sessions 2024 is coming up November 16-18 in Chicago, Illinois at McCormick Place Convention Center. Come a day early for Pre-Sessions Symposia, Early Career content, QCOR programming and the International Symposium on November 15. It's a special year you won't want to miss for the premier event for advancements in cardiovascular science and medicine as AHA celebrates its 100th birthday. Registration is now open, secure your spot here!When registering, use code NERDS and if you're among the first 20 to sign up, you'll receive a free 1-year AHA Professional Membership! Question #38 Mrs. M is a 65-year-old woman with non-ischemic dilated cardiomyopathy (LVEF 40%) and moderate to severe mitral regurgitation (MR) presenting for outpatient follow-up. Despite improvement overall, she continues to experience dyspnea on exertion with two flights of stairs and occasional PND. She reports adherence with her medication regimen of sacubitril-valsartan 97-103mg twice daily, metoprolol succinate 200mg daily, spironolactone 25mg daily, empagliflozin 10mg daily, and furosemide 80mg daily. A transthoracic echocardiogram today shows an LVEF of 35%, an LVESD of 60 mm, severe MR with a regurgitant fraction of 60%, and an estimated right ventricular systolic pressure of 40 mmHg. Her EKG shows normal sinus rhythm at 65 bpm and a QRS complex width of 100 ms. What is the most appropriate recommendation for management of her heart failure?AContinue maximally tolerated GDMT; no other changesBRefer for cardiac resynchronization therapy (CRT)CRefer for transcatheter mitral valve intervention Answer #38 ExplanationChoice C is correct. The 2020 ACC/AHA Guidelines for the management of patients with valvular heart disease outline specific recommendations.In patients with chronic severe secondary MR related to LV systolic dysfunction (LVEF
Despite advances in the management of acute myocardial infarction (MI), up to 38% of patients will experience signs of heart failure (HF) and many have a reduced left ventricular ejection fraction (LVEF). Sodium-glucose cotransporter 2 (SGLT2) inhibitors have well-documented cardiovascular benefits. However, data are limited regarding their use after an acute MI. Guest Authors: Amy Hu, PharmD and Kathleen Pincus, PharmD, BCPS, BCACP, CDCES Music by Good Talk
The following question refers to Section 7.4 of the 2022 AHA/ACC/HFSA Guideline for the Management of Heart Failure.The question is asked by the Director of the CardioNerds Internship Dr. Akiva Rosenzveig, answered first by Vanderbilt AHFT cardiology fellow Dr. Jenna Skowronski, and then by expert faculty Dr. Clyde Yancy.Dr. Yancy is Professor of Medicine and Medical Social Sciences, Chief of Cardiology, and Vice Dean for Diversity and Inclusion at Northwestern University, and a member of the ACC/AHA Joint Committee on Clinical Practice Guidelines.The Decipher the Guidelines: 2022 AHA / ACC / HFSA Guideline for The Management of Heart Failure series was developed by the CardioNerds and created in collaboration with the American Heart Association and the Heart Failure Society of America. It was created by 30 trainees spanning college through advanced fellowship under the leadership of CardioNerds Cofounders Dr. Amit Goyal and Dr. Dan Ambinder, with mentorship from Dr. Anu Lala, Dr. Robert Mentz, and Dr. Nancy Sweitzer. We thank Dr. Judy Bezanson and Dr. Elliott Antman for tremendous guidance.Enjoy this Circulation 2022 Paths to Discovery article to learn about the CardioNerds story, mission, and values. American Heart Association's Scientific Sessions 2024As heard in this episode, the American Heart Association's Scientific Sessions 2024 is coming up November 16-18 in Chicago, Illinois at McCormick Place Convention Center. Come a day early for Pre-Sessions Symposia, Early Career content, QCOR programming and the International Symposium on November 15. It's a special year you won't want to miss for the premier event for advancements in cardiovascular science and medicine as AHA celebrates its 100th birthday. Registration is now open, secure your spot here!When registering, use code NERDS and if you're among the first 20 to sign up, you'll receive a free 1-year AHA Professional Membership! Question #37 Mr. S is an 80-year-old man with a history of hypertension, type II diabetes mellitus, and hypothyroidism who had an anterior myocardial infarction (MI) treated with a drug-eluting stent to the left anterior descending artery (LAD) 45 days ago. His course was complicated by a new LVEF reduction to 30%, and left bundle branch block (LBBB) with QRS duration of 152 ms in normal sinus rhythm. He reports he is feeling well and is able to enjoy gardening without symptoms, though he experiences dyspnea while walking to his bedroom on the second floor of his house. Repeat TTE shows persistent LVEF of 30% despite initiation of goal-directed medical therapy (GDMT). What is the best next step in his management?AMonitor for LVEF improvement for a total of 60 days prior to further interventionBImplantation of a dual-chamber ICDCImplantation of a CRT-DDContinue current management as device implantation is contraindicated given his advanced age Answer #37 Explanation Choice C is correct. Implantation of a CRT-D is the best next step. In patients with nonischemic DCM or ischemic heart disease at least 40 days post-MI with LVEF ≤35% and NYHA class II or III symptoms on chronic GDMT, who have reasonable expectation of meaningful survival for >1 year,ICD therapy is recommended for primary prevention of SCD to reduce total mortality (Class 1, LOE A). A transvenous ICD provides high economic value in this setting, particularly when a patient's risk of death from ventricular arrhythmia is deemed high and the risk of nonarrhythmic death is deemed low. In addition, for patients who have LVEF ≤35%, sinus rhythm, left bundle branch block (LBBB) with a QRS duration ≥150 ms, and NYHA class II, III, orambulatory IV symptoms on GDMT, cardiac resynchronization therapy (CRT) is indicated to reduce total mortality, reduce hospitalizations, and improve symptoms and QOL. Cardiac resynchronization provides high economic value in this setting. Mr.
CardioNerds (Amit Goyal) join Dr. Merna Hussien, Dr. Akhil Kallur, Dr. Abhinav Saxena, and Dr. Brody Deb from the MedStar Georgetown - Washington Hospital Center in DC for a stroll around Rock Creek Park as they discuss an unusual case of cobalt cardiomyopathy. Expert commentary is provided by Dr. Nana Afari Armah. Episode audio was edited by CardioNerds Intern Christiana Dangas. The case is of a middle-aged woman with a past medical history of hypertension, hyperlipidemia, and bilateral hip replacements, who presented with subacute progressive exertional dyspnea, orthopnea, and constitutional symptoms and was found to have SCAI Stage C cardiogenic shock. Transthoracic echocardiogram showed severely reduced left ventricular ejection fraction (LVEF, 20-25%) and a moderate pericardial effusion. Cardiac catheterization revealed biventricular failure with elevated filling pressures. A cardiac MRI showed diffuse late gadolinium enhancement (LGE) in the left ventricle. Endomyocardial biopsy showed nonspecific chronic inflammation. However, the evidence of mitochondrial heavy metal toxicity and elevated cobalt levels made the diagnosis of cobalt cardiomyopathy. The patient underwent revision of hip joint implants to ceramic implants and started chelation therapy. However, due to persistent stage D heart failure despite normalization of cobalt levels, she underwent orthotropic heart transplantation. US Cardiology Review is now the official journal of CardioNerds! Submit your manuscript here. CardioNerds Case Reports PageCardioNerds Episode PageCardioNerds AcademyCardionerds Healy Honor Roll CardioNerds Journal ClubSubscribe to The Heartbeat Newsletter!Check out CardioNerds SWAG!Become a CardioNerds Patron! Case MEdia - Cobalt Cardiomyopathy Pearls - Cobalt Cardiomyopathy A good history goes a long way in diagnosing non-ischemic cardiomyopathy (NICM). Common problems can have uncommon presentations requiring a high degree of suspicion for diagnosis. Imaging features can overlap between causes of NICM. History helps in targeting further histological workup and uncovering the root cause. Multidisciplinary effort is essential in making a rare diagnosis. Taken from1 - Singh M, Krishnan M, Ghazzal A, Halushka M, Tozzi JE, Bunning RD, Rodrigo ME, Najjar SS, Molina EJ, Sheikh FH. From Hip to Heart: A Comprehensive Evaluation of an Infiltrative Cardiomyopathy. CJC Open. 2021 Nov 1;3(11):1392–5. Notes - Cobalt Cardiomyopathy How common is cobalt cardiomyopathy? When should it be suspected? Cobalt cardiomyopathy is incredibly rare, with only a handful of reported cases. 2 It is also known as beer drinkers' cardiomyopathy, as cobalt was added to beer for fortification in Quebec 3, where it was first reported. Cobalt cardiomyopathy is characterized by its rapidly progressive nature, the presence of low voltages on EKG, and diffuse infiltration. Patients also complained of a previous history of anorexia and weight loss and were found to have polycythemia and thyroid abnormalities on labs. This syndrome was very similar to wet beriberi except for the absence of a therapeutic response to thiamine. Taken from - 2 Later, this was noted in patients with total metal hip arthroplasty 4–6, especially in patients with metal-on-metal hip arthroplasty, which led to corrosion and leakage of cobalt into the bloodstream. The syndrome in these patients was similar to those in beer drinkers from Quebec. This figure, taken from 2, shows the reports of Cobalt cardiomyopathy after cobalt alloy prostheses. [HX1] What is the pathophysiology of cobalt cardiomyopathy? Cobalt has a variety of effects on the heart, both microscopically and biochemically.Cobalt may have multiple calcium-mediated cardiac effects and may also interfere with the Krebs cycle and ATP generation by mitochondria. Histology may show modest changes with no inflammatory response o...
The following question refers to Section 2.2 of the 2022 AHA/ACC/HFSA Guideline for the Management of Heart Failure.The question is asked by University of Colorado internal medicine resident Dr. Hirsh Elhence, answered first by University of Chicago advanced heart failure cardiologist and Co-Chair for the CardioNerds Critical Care Cardiology Series Dr. Mark Belkin, and then by expert faculty Dr. Mark Drazner.Dr. Drazner is an advanced heart failure and transplant cardiologist, Professor of Medicine, and Clinical Chief of Cardiology at UT Southwestern. He is the President of the Heart Failure Society of America.The Decipher the Guidelines: 2022 AHA / ACC / HFSA Guideline for The Management of Heart Failure series was developed by the CardioNerds and created in collaboration with the American Heart Association and the Heart Failure Society of America. It was created by 30 trainees spanning college through advanced fellowship under the leadership of CardioNerds Cofounders Dr. Amit Goyal and Dr. Dan Ambinder, with mentorship from Dr. Anu Lala, Dr. Robert Mentz, and Dr. Nancy Sweitzer. We thank Dr. Judy Bezanson and Dr. Elliott Antman for tremendous guidance.Enjoy this Circulation 2022 Paths to Discovery article to learn about the CardioNerds story, mission, and values. Question #35 A 50-year-old woman with a history of congestive heart failure, hypertension, type 2 diabetes mellitus, and obstructive sleep apnea presents to the outpatient clinic to follow up on her heart failure management. One year prior, echocardiogram showed an ejection fraction of 30% with an elevated BNP, for which she was started on appropriate GDMT. Repeat echocardiogram today showed an EF of 50%. Which of the following best describes her heart failure status? A HFrEF (HF with reduced EF) B HFimpEF (HF with improved EF) C HFmrEF (HF with mildly reduced EF) D HFpEF (HF with preserved EF) Answer #35 Explanation The correct answer is B – HFimpEF, or heart failure with improved ejection fraction, best describes her current heart failure status. Left ventricular ejection fraction is an important factor in classifying heart failure given differences in prognosis, response to treatment, and use in clinical trial enrollment criteria. The classification of heart failure by EF (adopted from the Universal Definition of HF): – HFrEF (HF with reduced EF): LVEF ≤40% – HFimpEF (HF with improved EF): previous LVEF ≤40%, a ≥10% increase from baseline LVEF, and a second measurement of LVEF >40%. – HFmrEF (HF with mildly reduced EF): LVEF 41%–49%, andevidence of spontaneous or provokable increased LV filling pressures (e.g., elevated natriuretic peptide, noninvasive and invasive hemodynamic measurement) – HFpEF (HF with preserved EF): LVEF ≥50%, and evidence of spontaneous or provokable increased LV filling pressures (e.g., elevated natriuretic peptide, noninvasive and invasive hemodynamic measurement) Patients with HFmrEF are usually in a dynamic state of improving from HFrEF or deteriorating towards HFrEF. Therefore, patients with HFmrEF may benefit from follow-up evaluation of systolic function and etiology of sub-normal EF. Improvements in EF are associated with better outcomes but do not indicate full myocardial recovery or normalization of LV function. Indeed, structural and functional abnormalities such as LV dilation and systolic or diastolic dysfunction often persist. Moreover, EF may remain dynamic with fluctuations in either direction depending on factors such as GDMT adherence and re-exposure to cardiotoxic agents. As such, the term heart failure with “improved EF” was deliberately chosen over “recovered EF” and “preserved EF”. Importantly, in patients with HFimpEF while on GDMT, the EF may decrease after withdrawal of GDMT. Main Takeaway
Circulation 68, No. 5, 939-950, 1983Background Coronary artery bypass surgery (CABG) had grown in popularity through the 1970s and 1980s. By 1981, approximately 159,000 bypass surgeries had been done.Cardiology Trial's Substack is a reader-supported publication. To receive new posts and support our work, consider becoming a free or paid subscriber.The goal of the CASS trial was to rigorously evaluate the effect of surgical vs medical therapy on total mortality in well-defined subsets of patients with coronary artery disease.Patients Eligible patients had to be 65 years of age or below and have angina that was Canadian Cardiovascular Society class I or II with or without a history of myocardial infarction, or had to have a well-documented MI more than 3 weeks before randomization. Clinical criteria for exclusion were prior CABG, unstable or progressive angina, angina more severe than class II (angina occurring after climbing one flight of stairs or walking two blocks is class III), congestive heart failure (New York Heart Association class III or IV), a coexisting illness that would increase the likelihood of death within 5 years, and a variety of practical exclusions that might limit active participation during follow-up.Angiographic requirements for participation in the trial included the presence of significant operable coronary artery disease, defined as either 70% or greater reduction in the diameter of the right, left anterior descending, or left circumflex coronary artery, or 50% or greater reduction in the diameter of the left main coronary artery. Patients with 70% or greater luminal diameter reduction of the left main were excluded. Also excluded were patients with LVEF (measured by left ventriculography) of less than 35% or those patients who required valve surgery or LV aneurysm repair.Baseline Characteristics There were 390 patients randomized in both the surgical and medical arm. The mean age was 51 years; 90% were males and 98% were White. Approximately 60% had previous MI, 30% had hypertension and only 3% had congestive heart failure. About 40% were smokers.Angiographic criteria were well matched—27% had one-vessel disease, 38% had two-vessel disease, and about a third had three-vessel disease. Nearly a third of patients in both groups had proximal LAD disease. The LVEF was more than 50% in 73-74% of patients in both groups. The LVEF was less than 50% in a fifth of patients in both arms.Procedures CASS authors were quite clear in the screening and randomization process. Slightly more than 16,600 patients were screened for participation in the trial at 11 centers. Figure 1 in the manuscript shows the reasons for exclusion, which included normal or minimal coronary disease (28%), Class 3 or 4 angina (36%), and left main disease more than 70%n (1.5%). Other exclusions totaled 16%.Ultimately there were 2099 patients eligible to be randomized, however, 1319 patients were not included in the trial due to physician preference.Randomization was stratified by clinical site, number of diseased vessels, and ejection fraction within three different clinical subgroups. Patients with angina and ejection fractions of at least 0.50 were randomized within group A (n =514), those with angina and EF less than 0.50 were randomized within group B (n=106) and those free of angina after well-documented MI were randomized within group C (n=160).A total of 954 distal anastomoses were constructed in 357 patients in the surgical group, an average of 2.7 per patient. A total of 334 distal anastomoses were evaluated in the 129 patients; 90% of the grafts were open, 97% of patients had at least one open graft, and in 81% of patients all grafts were patent. Medical therapy consisted mostly of nitrates, and beta-blockers. Statin drugs were not available during the trial. Endpoint The primary endpoint was all-cause mortality. It was assessed with the intention-to-treat method. Results There were no significant differences in mortality.At 5 years, the average annual mortality rate in patients assigned to surgical treatment was 1.1%. The annual mortality rate in those receiving medical therapy was 1.6%. Expressed differently, the rate of death at 5 years was 9.2% in the medical group versus 7.4% in the surgical group. Annual mortality rates in patients with single-, double-, and triple-vessel disease who were in the surgical group were 0.7%, 1.0%, and 1.5%; the corresponding rates in patients in the medical group were 1.4%, 1.2%, and 2.1%.There were also no significant differences in survival when patients were grouped according to degree of coronary artery disease (number of vessels) or EF or by a combination of diseased vessels and EF.Crossovers did occur. Approximately 23% of the 390 patients assigned to the medical group had surgery during the follow-up period (4.7% per year). Of the patients assigned to surgery, 31 of 390 patients (8%) did not have surgery.Conclusions The CASS authors write in the abstract of the manuscript that they observed excellent long-term survival in both groups and that for patients similar to those in the trial, surgery could be deferred until symptoms worsen.The CASS trial had caveats. First was that the 780 patients enrolled in the trial were highly selected from a total of more than 16,000 patients in the registry. The CASS registry revealed widely disparate annual mortality in patients managed medically, ranging from 1.3% for those with single-vessel disease and normal EF to 12.5% in those with three-vessel disease and impaired EF. Another caveat was the lower-than-expected annual mortality in the medical arm of only 1.6%. This was lower than previous surgery trials (3.3% noted in the European Collaborative Study and 4.3% reported in the Veterans Administration Study). CASS authors estimated a 2-4% annual mortality. This reduces the power to find differences in the two groups.It is interesting however, that for the 1319 patients in whom their physician declined randomization, the outcome in those treated medically was similar to that reported in the randomly assigned patients who received medical therapy.In conclusion, as early as the 1980s, the CASS study showed that stable coronary artery disease was quite stable, and that revascularization in selected patients did not improve survival over medical therapy. Cardiology Trial's Substack is a reader-supported publication. To receive new posts and support our work, consider becoming a free or paid subscriber. Get full access to Cardiology Trial's Substack at cardiologytrials.substack.com/subscribe
CardioNerds co-founder Dan Ambinder joins Dr. Lefan He, Dr. Sina Salehi Omran, and Dr. Neil Gupta from the University of Rochester Cardiovascular Disease Fellowship Program for a day sailing on Lake Ontario. Expert commentary is provided by Dr. Jeffrey Bruckel, and CV Fellowship Program Director Dr. Burr Hall shares insights on the University of Rochester fellowship. The episode audio was edited by CardioNerds intern Dr. Atefeh Ghorbanzadeh. They discuss the following case involving a patient with papillary muscle rupture. This is a 63-year-old man with hypertension, hyperlipidemia, and active tobacco smoking who presented with acute dyspnea. He was tachycardic but otherwise initially hemodynamically stable. The physical exam demonstrated warm extremities with no murmurs or peripheral edema. Chest X-ray revealed diffuse pulmonary edema, and the ECG showed sinus tachycardia with T-wave inversions in the inferior leads. A bedside echocardiogram revealed a flail anterior mitral valve leaflet. The patient was taken for cardiac catheterization that revealed nonobstructive mid-RCA atheroma with a distal RCA occlusion, which was felt to reflect embolic occlusion from recanalized plaque. PCI was not performed. Right heart catheterization then demonstrated a low cardiac index as well as elevated PCWP and PA pressures. An intra-aortic balloon pump was placed at that time. A TEE was performed soon after which showed the posteromedial papillary muscle was ruptured with flail segments of the anterior mitral leaflet as well as severe posteriorly directed mitral regurgitation. The patient ultimately underwent a successful tissue mitral valve replacement and CABG. US Cardiology Review is now the official journal of CardioNerds! Submit your manuscript here. CardioNerds Case Reports PageCardioNerds Episode PageCardioNerds AcademyCardionerds Healy Honor Roll CardioNerds Journal ClubSubscribe to The Heartbeat Newsletter!Check out CardioNerds SWAG!Become a CardioNerds Patron! case Media Pearls - A Case of Papillary Muscle Rupture Most cases of papillary muscle rupture demonstrate only small areas of ischemia with preserved ventricular function, thus causing high shear force on the ischemic papillary muscle. The posteromedial papillary muscle has a single blood supply from the posterior descending artery, while the anterolateral papillary muscle has a dual blood supply from the LAD and the circumflex. Therefore, the posteromedial papillary muscle is more vulnerable to ischemia and, hence, rupture. A murmur may be absent in cases of papillary muscle rupture due to the rapid equalization of left atrial and left ventricular pressures caused by the acuteness of the severe MR. Papillary muscle rupture should always be on the differential for acute dyspnea when ACS is suspected. While mostly associated with STEMIs, mechanical complications of acute myocardial infarctions can also occur after NSTEMIs. Always auscultate patients carefully after a myocardial infarction! When evaluating patients with chest pain presenting with acute or rapidly progressive heart failure and a hypercontractile LVEF should raise suspicion for mechanical complications of MI. Once a papillary muscle rupture is diagnosed, cardiac surgery should be immediately contacted. Temporizing measures prior to surgery include positive pressure ventilation, IV nitroglycerin/nitroprusside, and temporary mechanical circulatory support. Notes - A Case of Papillary Muscle Rupture What is the clinical presentation of acute mitral regurgitation from papillary muscle rupture? Patients typically present 3-5 days after a transmural infarct. Roughly half of these patients present with pulmonary edema that may quickly progress to cardiogenic shock. Most cases are associated with STEMIs, but papillary muscle rupture is also possible with an NSTEMI.
N Engl J Med 2024;390:1455-1466BackgroundDespite advances in the care of patients after myocardial infarction, there remains residual risk of heart failure and death. The amount of risk parallels the degree of left ventricular systolic dysfunction. Previous studies have shown that the drug class of sodium–glucose cotransporter 2 (SGLT2) inhibitors reduce cardiovascular risk (especially recurrent heart failure) in multiple clinical situations.The goal of the placebo-controlled Empagliflozin on Hospitalization for Heart Failure and Mortality in Patients with Acute Myocardial Infarction (EMPACT-MI) trial was to determine whether empagliflozin reduced the risk of heart failure or death in patients with acute MI and either a new reduction in LV function or signs of congestion, or both.PatientsAdult patients who had been hospitalized with MI within 14 days before randomization. There had to be a new LVEF 65 (50%), Type 2 DM (32%) and 3-vessel CAD (31%). Slightly more than 70% of patients had more than one enrichment factor.Approximately 20% of patients had an LVEF > 45%. Slightly more than half of patients had an LVEF between 35% and 45%. Trial ProceduresRandomization was 1:1 to empagliflozin 10mg daily or matching placebo. The trial was conducted between 2020-2023 at 451 sites in 22 countries. The median time from admission to randomization was 5 days. The trial had a streamlined design, with the collection of essential data only, including information about specific safety events, and mainly remote follow-up of patients (by means of a Web-based application or a telephone call) with only a few face-to-face visits; the trial assessed investigator-reported end-point events rather than centrally adjudicated end-point events. Specifically, follow up included a remote visit at 2 weeks, a face-to-face visit at 6 months, and remote visits every 6 months thereafter until the end of the trial, when a final telephone call was performed.EndpointsThe primary end point was a composite of hospitalization for heart failure or death from any cause as assessed in a time-to-first-event analysis.The key secondary end points in the prespecified hierarchical testing strategy were the total number of hospitalizations for heart failure or death from any cause, the total number of nonelective cardiovascular hospitalizations or death from any cause, the total number of nonelective hospitalizations for any cause or death from any cause, and the total number of hospitalizations for myocardial infarction or death from any cause. Trial authors estimated that 532 patients with a primary end-point event would provide the trial with 85% power to detect a 23% lower risk of an event in the empagliflozin group than in the placebo group, with a two-sided type I error of 0.05. However, the trial originally planned to enroll about 3300 patients, with the option to enroll 5000 patients. But then the trial was further increased to 6500 patients.Key secondary endpoints were assessed using a prespecified hierarchical testing procedure. This began with the primary endpoint.ResultsAn interesting aspect of this trial, and more recent post-MI trials is that the ratio of screened to enrolled patients is almost 1:1. Whereas older trials screened many more patients than were enrolled, in EMPACT, only 88 of 6600 screened patients were excluded. In total, approximately 3200 patients were randomized in both arm.After a median follow-up of 18 months, a primary end-point event — a first hospitalization for heart failure or death from any cause — occurred in 8.2% in the empagliflozin group and in 9.1% in the placebo group, with incidence rates of 5.9 and 6.6 events, respectively, per 100 patient-years (hazard ratio, 0.90; 95% confidence interval [CI], 0.76 to 1.06; P=0.21). A look at the two components showed that there were fewer heart failure hospitalizations in the empagliflozin group (3.6% vs 4.7%; HR 0.77 (0.60–0.98)) Overall death were similar in both groups. As for secondary endpoints, total heart failure hospitalizations was 2.4 vs 3.6 events, respectively, per 100 patient-years (rate ratio, 0.67; 95% CI, 0.51 to 0.89). The composite of total heart failure hospitalizations or death were not significantly different (HR 0.87 0.68-1.10). CV death was 4% in both groups. There were no obvious subgroup effects nor differences in safety. Conclusions The addition of empagliflozin did not significantly reduce a composite endpoint of heart failure admissions and death. Even the reduction in heart failure admissions was modest. Given the medication burden of the typical patient after myocardial infarction, coupled with the high cost of this drug class, we see no strong evidence for routine use of SGLT2i in EMPACT-MI-type patients. The null results of this trial and PARADISE-MI speak to the beneficial effects of modern post-MI care—including mostly rapid revascularization with PCI. It is difficult to improve on baseline care of the MI patient in 2024. Cardiology Trial's Substack is a reader-supported publication. To receive new posts and support our work, consider becoming a free or paid subscriber. Get full access to Cardiology Trial's Substack at cardiologytrials.substack.com/subscribe
NEJM 2021:385:1845-55.Background Over two decades had passed since the publications of the seminal trials comparing ACE inhibitors with placebo in post-MI patients with LV dysfunction and congestive heart failure (SAVE, AIRE and TRACE). The VALIANT trial, published in 2003, found that the ARB drug Valsartan was as effective as Captopril in improving survival and reducing cardiovascular morbidity in this patient population. Thus, for many years, a cornerstone of managing post-MI patients with LV dysfunction and heart failure involved afterload reduction with ACE inhibitors or ARBs.Then in 2014, the landscape of heart failure management changed with the publication of the PARADIGM-HF trial which found that Entresto, a drug combining the ARB Valsartan and the neprilysin inhibitor Sacubitril, significantly reduced death and heart failure hospitalizations. We will review PARDIGM-HF later since it enrolled patients with chronic, stable heart failure and not post-MI patients. The rationale for the combination drug is that it simultaneously blocks the renin-angiotensin system and inhibits of the breakdown of several vasoactive peptides including atrial natriuretic peptide (ANP) and brain natriuretic peptide (BNP), serving to enhance vasodilation and reduce blood volume.The PARADISE-MI trial sought to test the hypothesis that early initiation of Sacubitril-Valsartan in post-MI patients with LV dysfunction and congestive heart failure would reduce cardiovascular death or incident heart failure compared to the ACE inhibitor Ramipril.Cardiology Trial's Substack is a reader-supported publication. To receive new posts and support our work, consider becoming a free or paid subscriber.Patients Adults without a history of heart failure who had a spontaneous MI within 0.5 to 7 days before presentation who had either a LVEF of /= 70 years, diabetes, previous MI, eGFR
William H. Sauer, MD, FHRS, CCDS, Brigham and Women's Hospital is joined by Edward P. Gerstenfeld, MD, MS, FHRS, University of California, San Francisco, and Yasser Rodriguez, MBA, MD, Cleveland Clinic Florida to discuss; Background: Premature ventricular complexes (PVCs) are common and associated with worse outcomes in patients with heart failure. Class 1C antiarrhythmic drugs (AADs) effectively suppress PVCs, but guidelines currently restrict their use in structural heart disease. Conclusions: Class 1C AADs effectively suppressed PVCs in patients with NICM and ICDs, leading to increases in LVEF and biventricular pacing percentage. In this limited sample, their use was safe. Larger studies are needed to confirm the safety of this approach. https://www.hrsonline.org/education/TheLead https://doi.org/10.1016/j.jacep.2024.01.021 Host Disclosure(s): W. Sauer: Honoraria/Speaking/Consulting: Biotronik, Biosense Webster, Inc., Abbott, Boston Scientific, Research: Medtronic Contributor Disclosure(s): E. Gerstenfeld: Honoraria/Speaking/Teaching: Abbott, Biosense Webster, Inc., Boston Scientific, Medtronic, Membership on Committees/Advisory Boards: Boston Scientific, Farapulse, Honoraria/Speaking/Consulting: Boston Scientific, Abbott Medical, Medtronic, Adagio Medical, Research: Boston Scientific Y. Rodriguez: Nothing to disclose. This episode has .25 ACE credits associated with it. If you want credit for listening to this episode, please visit the episode page on HRS365 https://www.heartrhythm365.org/URL/TheLeadEpisode66
CardioNerds (Dr. Jessie Holtzman, Chair for the CardioNerds Women's Heart Disease Committee, and Dr. Naima Maqsood, Chair for the CardioNerds Electrophysiology Committee) join Dr. Ritika Gadodia, Dr. Namratha Meda, and Dr. Tsion Aberra from the Medstar Washington Hospital Center/Georgetown University Program for the National Cherry Blossom Festival. They discuss involving a patient with Chagas cardiomyopathy. Dr. Rachel Marcus provides the Expert CardioNerd Perspectives & Review segment for this episode. Episode audio was edited by Dr. Diane Masket. A 79-year-old male with a history of cardiomyopathy presented with recurrent ventricular tachycardia (VT) post-CRT-D placement. On arrival, the patient was in cardiogenic shock. Initial treatment with amiodarone and milrinone failed, necessitating the addition of mexiletine. Imaging was suggestive of a left ventricular ejection fraction of 20-25% with severe global hypokinesis. Prior coronary angiogram had shown nonobstructive coronary artery disease. Further non-ischemic cardiomyopathy evaluation was unrevealing. Given his El Salvadorian origins, Chagas serology results revealed Chronic Chagas Cardiomyopathy (CCM) confirmed by CDC testing. This case underscores the importance of suspecting CCM in patients with risk factors. An early diagnosis of CCM, can prevent catastrophic events (heart blocks, ventricular arrhythmias, thromboembolic events). In summary, this case takes the learner through the journey of a patient with non-ischemic cardiomyopathy and emphasizes the importance of approaching it with a wide range of differentials. US Cardiology Review is now the official journal of CardioNerds! Submit your manuscript here. CardioNerds Case Reports PageCardioNerds Episode PageCardioNerds AcademyCardionerds Healy Honor Roll CardioNerds Journal ClubSubscribe to The Heartbeat Newsletter!Check out CardioNerds SWAG!Become a CardioNerds Patron! Case Media Pearls - Chronic Chagas Cardiomyopathy with Recurrent Ventricular Tachyarrhythmia Always consider Chagas cardiomyopathy when you have a patient from Latin America who presents with non-ischemic cardiomyopathy. Chagas cardiomyopathy is associated with an unfavorable prognosis and serves as an independent predictor of mortality. Chagas cardiomyopathy is arrhythmogenic and requires consideration for ICD and, when appropriate, catheter based ventricular tachycardia ablation. It is crucial to treat patients with nifurtimox and benznidazole when appropriate. Provide screening for first-degree family members or close relatives who may have lived in the same environment. Show Notes - Chronic Chagas Cardiomyopathy with Recurrent Ventricular Tachyarrhythmia What is the disease progression in Chagas disease5? Acute Stage:Initial infection occurs through contact with infected triatomine bug feces or contaminated blood products.Symptoms may be mild or absent but can include fever, fatigue, body aches, and swelling at the injection site (chagoma). Parasitemia is high during this stage. Intermediate/Indeterminate Stage:The infection becomes chronic if left untreated.Many individuals enter this stage with no noticeable symptoms.Parasitemia levels decrease, but the parasite remains in the body, mainly in muscle and cardiac tissue. This stage can last for years to decades. Chronic Stage:Some individuals will remain asymptomatic throughout their lives.Cardiac complications (chronic Chagas cardiomyopathy) can lead to arrhythmias, congestive heart failure, and sudden death. Digestive complications can result in enlarged esophagus (megaesophagus) and colon (megacolon), leading to difficulties in swallowing and digestion. When do we suspect, and who do we screen, for Chagas disease? The seroprevalence of CCM in the USA is as high as 19%16. Among patients with LVEF
First prescription smartphone app approved for major depressive disorder; Novel blood test to detect TBI; Zevtera approved for bloodstream infections; Mpox vaccine is now commercially available; Algorithm cleared for LVEF dectection.
Dr. Aubry DeGrey Longevity specialist, author of Ending Aging and founder of LVEF.org explains to me the recent advances in allowing people to live longer and talks about some current projects. Please take a listen to this important message. @aubreydegrey @LongevityEVF http://levf.org/donate https://www.amazon.com/Ending-Aging-Rejuvenation-Breakthroughs-Lifetime/dp/0312367074
Dr. Aubry DeGrey Longevity specialist, author of Ending Aging and founder of LVEF.org explains to me the recent advances in allowing people to live longer and talks about some current projects. Please take a listen to this important message. @aubreydegrey @LongevityEVF http://levf.org/donate https://www.amazon.com/Ending-Aging-Rejuvenation-Breakthroughs-Lifetime/dp/0312367074Become a supporter of this podcast: https://www.spreaker.com/podcast/heartland-newsfeed-radio-network--2904397/support.
The Western AF meeting, aspirin, cannabis use, LVEF in athletes, and shared decision making before ICD implantation are the topics John Mandrola, MD, discusses in this week's podcast. This podcast is intended for healthcare professionals only. To read a partial transcript or to comment, visit: https://www.medscape.com/twic I. ASA in Primary Prevention Campbell Meta-analysis https://www.ahajournals.org/doi/10.1161/CIRCULATIONAHA.123.065420 Swedish Observational study https://www.ahajournals.org/doi/10.1161/CIRCULATIONAHA.117.028321 ASPREE https://www.nejm.org/doi/full/10.1056/NEJMoa1805819 II. Cannabis Cannabis Use Tied to Increased Cardiovascular Risk https://www.medscape.com/viewarticle/cannabis-use-tied-increased-cardiovascular-risk-2024a10003yr It Sure Looks Like Cannabis Is Bad for the Heart, Doesn't It? https://www.medscape.com/viewarticle/1000250 Journal of the AHA Observational Study https://www.ahajournals.org/doi/full/10.1161/JAHA.123.030178 III. Low EF in Athletes Reduced Ejection Fraction in Elite Endurance Athletes: Clinical and Genetic Overlap With Dilated Cardiomyopathy https://www.ahajournals.org/doi/10.1161/CIRCULATIONAHA.122.063777 IV. ICDs and Shared Decision-Making Association of a Medicare Mandate for Shared Decision-Making With Cardiac Device Utilization https://jamanetwork.com/journals/jamainternalmedicine/article-abstract/2815017 You may also like: The Bob Harrington Show with the Stephen and Suzanne Weiss Dean of Weill Cornell Medicine, Robert A. Harrington, MD. https://www.medscape.com/author/bob-harrington Questions or feedback, please contact news@medscape.net
“The search for daunorubicin's sister really led to this discovery of doxorubicin, which is an analog with much greater activity. The discovery of doxorubicin can be coined kind of as, ‘one of the best drugs born in Milan, Italy.' And after that, a few analogs were developed and tested, and two that we currently use today, are idarubicin and epirubicin,” Puja Patel, PharmD, BCOP, clinical oncology pharmacist at the Delnor Hospital Northwestern Medicine Cancer Center in Geneva, IL, told Jaime Weimer, MSN, RN, AGCNS-BS, AOCNS®, manager of oncology nursing practice at ONS, during a discussion about anthracyclines and other antitumor antibiotics. This episode is part of a series about drug classes, which we'll include a link to in the episode notes. You can earn free NCPD contact hours after listening to this episode and completing the evaluation linked below. Music Credit: “Fireflies and Stardust” by Kevin MacLeod Licensed under Creative Commons by Attribution 3.0 Earn 0.75 contact hours of nursing continuing professional development (NCPD), which may be applied to the nursing practice, oncology nursing practice, symptom management, palliative care, supportive care, or treatment ILNA categories, by listening to the full recording and completing an evaluation at myoutcomes.ons.org by January 26, 2026. The planners and faculty for this episode have no relevant financial relationships with ineligible companies to disclose. ONS is accredited as a provider of NCPD by the American Nurses Credentialing Center's Commission on Accreditation. Learning outcome: The learner will report an increase in knowledge of anthracyclines and antitumor antibiotics. Episode Notes Complete this evaluation for free NCPD. Oncology Nursing Podcast: Pharmacology 101 series ONS Voice oncology drug reference sheets IV Cancer Treatment Education Sheets ONS Voice articles: The Evidence Is Building for ACE Inhibitors in Anthracycline-Associated Cardiotoxicity Outpatient Oncology Drug Series: Doxorubicin Is the Infamous Red Devil Clinical Journal of Oncology Nursing articles: Nursing Alchemy: Transforming R-CHOP Information Into Essentials Dyspnea: Common Side Effect Cardiac Toxicity: Using Angiotensin-Converting Enzyme Inhibitors to Prevent Anthracycline-Induced Left Ventricular Dysfunction and Cardiomyopathy Oncology Nursing Forum article: Symptom Clusters in Lymphoma Survivors Before, During, and After Chemotherapy: A Prospective Study ONS Huddle Card: Antitumor Antibiotics Additional healthcare professional resources: Blindspot: Hidden Biases of Good People Harvard University Implicit Association Test OncoPharm Podcast ASCO Education Podcast Additional patient resources: National Comprehensive Cancer Network patient resources National Comprehensive Cancer Network patient webinars National Cancer Institute resources for patients To discuss the information in this episode with other oncology nurses, visit the ONS Communities. To find resources for creating an ONS Podcast Club in your chapter or nursing community, visit the ONS Podcast Library. To provide feedback or otherwise reach ONS about the podcast, email pubONSVoice@ons.org. Highlights From Today's Episode “Anthracyclines are kind of categorized as topoisomerase II inhibitors, and these agents are very powerful in that they have—it's really like three drugs in one—they have various mechanisms.” TS 3:55 “We need to create a stable environment, and so we actually cut one of the cords, and that's exactly what topoisomerase is doing. It's cutting one of the DNA strands. And in this case, it's cutting two strands, and that's why it's called topoisomerase II, so it's cutting both of the strands. It's cutting the DNA, releasing some of that tension, allowing for replication, and then rejoining that portion. So, it's a very important enzyme, and it'll go about doing this for the entire strand of DNA.” TS 4:50 “The other second mechanism is kind of the effect on DNA. So, you'll come across reading the term ‘DNA intercalation.' So, what does that word mean? When you take the word ‘intercalate,' the definition of it means ‘intrusive inserting of something in an existing series or sequence.' The analogy that I could think of here is simple: It's thinking about too many passengers squeezing in the backseat of your car. There could be safety issues, there's weight issues, there's instability maybe while driving. And that's what this doxorubicin is doing. It's sliding right in between the base pairs of the DNA double helix, destroying hydrogen bonds between those two bases, which then change the shape of that double helix. And by changing the shape, topoisomerase II, which we just talked about, can no longer go in and bind to DNA. It can't relax that super coil. And so, DNA synthesis doesn't happen.” TS 6:02 “So, the main toxicity that our listeners might be familiar with is cardiotoxicity. And also with cardiotoxicity, breaking it down a little bit, there's an onset that occurs during treatment or even years to decades, and that's kind of this delayed cardiotoxicity. Signs and symptoms of acute cardiotoxicity could vary from EKG changes present as tachycardia, tachyarrhythmia. Delayed cardiotoxicity is anything from heart failure to left ventricular ejection fraction decrease.” TS 9:41 “We're worried about heart failure in these patients. So, we might see EKG changes, we might see LVEF [left ventricular ejection fraction] changes, and we're kind of tracking these agents based on what is called cumulative dose tracking or lifetime dose. So, all of these agents have specific lifetime maximums that we need to be aware of.” TS 14:53 “So, smoking, hypertension, diabetes, dyslipidemia, obesity, or you're older in age, or perhaps you have a compromised cardiac function—you're at greater risk for developing these cardiotoxicities. An example that I've had in my clinic is I've identified some of these patients that have these risk factors, and we go into a little bit more aggressive monitoring for the echocardiogram or MUGA [multigated acquisition]. And when we put in those orders, we often get denials from insurance. We submit the guidelines in, kind of, appeals to help those patients kind of proactively realize if we're putting them in a greater cardiac risk.” TS 15:47 “One of the biggest things is for nurses to kind of look over their policies for administration for vesicants and specifically checking blood return for these agents, because many of them are given, you know, IV push. So, checking blood return every 2–5 ml is really important to make sure that you are in the right space. And then these agents, some of them can also be given continuously. So, you're thinking about, first of all, you should have a central line in for these agents because they're vesicants. But if it's being given continuous, there is something that's called anthracycline streaking, and it's not the same as an extravasation. So, I think being able to decipher the difference between the two is really, kind of, comes with experience.” TS 20:36 “I think awareness is really essential. And thankfully, you know, thankfully or not, I guess, you were with the patient for this entire time, right? Because you're pushing every 2–5 ml, you're checking. So, it's a very kind of intimate experience in and of itself. So, I think just being very vigilant is very important.” TS 22:24 “So, to talk about bleomycin here, for example, kinetically, two-thirds of this drug is eliminated renally. And so, we would think that there would need to be renal adjustments if there's renal changes. So, for creatinine clearance greater than 50, there are no renal dose adjustments. But after that, every 10 ml per minute decrease in GFR [glomerular filtration rate], there are dose reductions that are required. And this drug, in particular, has a lot of gradations in terms of renal dysfunction that I've seen.” TS 27:30 “Thinking about bleomycin, it's IV over 10 minutes, and you want to think about the lifetime maximum dose. So, when you are working up your patient, that's something to kind of think about. Dactinomycin is highly emetogenic, so making sure that there's antibiotics on board. It's also a vesicant, so thinking about vesicants precautions. Cold compresses is how you would help treat that if there is an extravasation.” TS 33:14 “I think trust is the foundation oncology really because we are asking our patients to do so many things outside of our infusion center, picking up medications, taking medications, calling us about signs and symptoms, going and getting all these imaging know. So, if there isn't that foundation of trust, having this perfect curative treatment plan may be more challenging to really be carried out.” TS 38:06 “We've developed these very powerful agents, and they're non–cell specific. So, I think the next step would be, how can we reformulate them to make them less toxic and provide more of a targeted approach? And so, perhaps an antibody-drug conjugate that is specifically attacking the lymphoma or the breast cell can deliver this chemotherapy with a cytotoxic payload is there in the horizon.” TS 39:07 “I think the misconception that ‘I will develop heart damage' is really important. Doxorubicin has the infamous name of the red devil, but I think it's important to let your patients know that heart failure increases with cumulative dosing. You know, talking to them about 300 mg/m2 is associated with a 1.5% heart failure risk. Whereas going all the way across to 500 mg/m2, now you're looking at 6%–20% probability of developing heart failure.” TS 42:30 “I think taking the time and understanding the literature. Typically, we don't start these agents with LVEF less than 50–55. There's some great review articles in JCO [Journal of Clinical Oncology] that kind of define what cardiomyopathy decrease looks like and decreases in LVEF over 10% to a value below the institutional limit of normal, I think, is a nice point to have as a value, a number to kind of work with.” TS 43:53 “Working with your nurse educator and leader to help achieve OCN®, oncology certified nurse, certification is really important. And I think live simulated experiences are really beneficial, maybe even looking at extravasations or having an infusion-related reaction, because here in the acute setting, we're really kind of in this like responsive mode. But if we practice, we can respond more deliberately and more calmly.” TS 45:05
Lancet 1990;336:65-71.Background Large trials up to this point had established the role of thrombolytic therapy and aspirin in patients with acute MI. The next question centered on the different types of thrombolytic agents as well as the merits of adding high dose heparin to aspirin following revascularization. Data from smaller studies showed that alteplase (tPA) had a higher recanalization rate at 90 minutes compared to streptokinase (SK). “Recanalization” is a surrogate endpoint – a person doesn't necessarily care whether their artery is open or not at 90 minutes, but instead, whether they live or die, and how they live in the aftermath of a heart attack. The GISSI-2 trial sought to test the hypothesis that tPA would reduce the composite hard endpoint of mortality and extensive LV dysfunction compared to streptokinase (SK) and that heparin plus aspirin compared to aspirin alone would do the same.Cardiology Trial's Substack is a reader-supported publication. To receive new posts and support our work, consider becoming a free or paid subscriber.Patients Patients were eligible who presented with chest pain and ST segment elevations of ≥1 mm in any limb lead of the ECG and/or ≥2 mm in any precordial leads and were admitted to the CCU within 6 hr from the onset of symptoms. Absolute contraindications included: recent or current bleeding, stroke within the previous 6 months, a surgical or invasive procedure or trauma within the previous 2 weeks, uncontrolled hypertension defined as SBP ≥200 mmHg or DBP ≥110 mmHg, or previous treatment with SK within 6 months.Baseline characteristics The number of patients who were admitted to CCUs with acute myocardial infarctions (MI) over the study period was 38,086 and 12,490 (33%) were ultimately enrolled. Reasons for exclusion included: more than 6 hr from onset of symptoms (61%), contraindications to fibrinolysis (15%), unlikely to have acute MI (13%), administrative reasons (5%), ST depressions (4%) and 1% were not reported. Similar to GISSI-1, which also provided information on patients enrolled versus those who were not, women were more likely to be excluded (30% of excluded vs 20% of enrolled) as were patients >70 years of age (41% of excluded vs 22% of enrolled). The death rate of those excluded was higher (11% of excluded vs 9% of enrolled).Approximately 80% of patients enrolled were men under the age of 70. Patients with inferior (34%) and anterior STEMI's (31%) composed more than half of the cohort. 72% of patients presented within 3 hours of symptom onset and over 95% had Killip scores of II or below with the vast majority being Killip I (78%).Procedures Immediately following randomization half of all patients received either 1.5 MU of SK infused over 1 hour or 100 mg of tPA infused over 3 hours. Since this was a 2x2 factorial design, half of all patients also received 12,500 U of subcutaneous heparin twice daily starting 12 hours after the beginning of the tPA or SK infusion and to be continued until hospital discharge.All patients without specific contraindications were recommended to receive oral aspirin 325 mg/day and 5-10 mg of IV atenolol, even before randomization, as soon as evolving MI was diagnosed.Endpoints The combined primary endpoint consisted of all-cause mortality plus the number of patients who had late (day 4 or later) clinical congestive heart failure, or extensive LV damage (LVEF
Go online to PeerView.com/UBM860 to view the activity, download slides and practice aids, and complete the post-test to earn credit. Hypertrophic cardiomyopathy (HCM) is the most common inherited cardiac disease, with a mortality rate estimated to be about three times greater than that experienced by individuals of similar age without HCM. So far, the FDA has approved only one noninvasive treatment option indicated for the treatment of HCM—a first-in-class cardiac myosin inhibitor (CMI) called mavacamten. A second CMI, aficamten, is currently being studied. As timely diagnosis of patients with HCM may reduce or delay the need for invasive treatment, these treatment options are essential for cardiologists to understand. This PeerView activity covers what you need to know about diagnosing and managing HCM for optimal patient care. A patient case threads through each section and offers a window into real-world diagnostic and treatment challenges as our expert shares guidance and the latest evidence. By the end of this activity, you will be able to explain the underlying pathophysiology of HCM, apply your new knowledge for timely diagnosis and individualized treatment of HCM, and implement strategies for long-term patient safety and improved outcomes. Upon completion of this activity, participants should be better able to: Apply current guidance and the latest evidence to support accurate differential diagnosis and encourage early treatment in patients suspected of having HCM; Evaluate the efficacy and safety evidence supporting the ability of modern targeted strategies to address the underlying pathophysiology of HCM; Individualize treatment of HCM based on patients' personal preferences and goals of care and identify opportunities to reduce inequities; and Implement strategies that encourage and support regular assessment of clinical status, LVEF, and LVOT gradient in patients with HCM, consulting with specialty colleagues as needed.
Go online to PeerView.com/UBM860 to view the activity, download slides and practice aids, and complete the post-test to earn credit. Hypertrophic cardiomyopathy (HCM) is the most common inherited cardiac disease, with a mortality rate estimated to be about three times greater than that experienced by individuals of similar age without HCM. So far, the FDA has approved only one noninvasive treatment option indicated for the treatment of HCM—a first-in-class cardiac myosin inhibitor (CMI) called mavacamten. A second CMI, aficamten, is currently being studied. As timely diagnosis of patients with HCM may reduce or delay the need for invasive treatment, these treatment options are essential for cardiologists to understand. This PeerView activity covers what you need to know about diagnosing and managing HCM for optimal patient care. A patient case threads through each section and offers a window into real-world diagnostic and treatment challenges as our expert shares guidance and the latest evidence. By the end of this activity, you will be able to explain the underlying pathophysiology of HCM, apply your new knowledge for timely diagnosis and individualized treatment of HCM, and implement strategies for long-term patient safety and improved outcomes. Upon completion of this activity, participants should be better able to: Apply current guidance and the latest evidence to support accurate differential diagnosis and encourage early treatment in patients suspected of having HCM; Evaluate the efficacy and safety evidence supporting the ability of modern targeted strategies to address the underlying pathophysiology of HCM; Individualize treatment of HCM based on patients' personal preferences and goals of care and identify opportunities to reduce inequities; and Implement strategies that encourage and support regular assessment of clinical status, LVEF, and LVOT gradient in patients with HCM, consulting with specialty colleagues as needed.
Go online to PeerView.com/UBM860 to view the activity, download slides and practice aids, and complete the post-test to earn credit. Hypertrophic cardiomyopathy (HCM) is the most common inherited cardiac disease, with a mortality rate estimated to be about three times greater than that experienced by individuals of similar age without HCM. So far, the FDA has approved only one noninvasive treatment option indicated for the treatment of HCM—a first-in-class cardiac myosin inhibitor (CMI) called mavacamten. A second CMI, aficamten, is currently being studied. As timely diagnosis of patients with HCM may reduce or delay the need for invasive treatment, these treatment options are essential for cardiologists to understand. This PeerView activity covers what you need to know about diagnosing and managing HCM for optimal patient care. A patient case threads through each section and offers a window into real-world diagnostic and treatment challenges as our expert shares guidance and the latest evidence. By the end of this activity, you will be able to explain the underlying pathophysiology of HCM, apply your new knowledge for timely diagnosis and individualized treatment of HCM, and implement strategies for long-term patient safety and improved outcomes. Upon completion of this activity, participants should be better able to: Apply current guidance and the latest evidence to support accurate differential diagnosis and encourage early treatment in patients suspected of having HCM; Evaluate the efficacy and safety evidence supporting the ability of modern targeted strategies to address the underlying pathophysiology of HCM; Individualize treatment of HCM based on patients' personal preferences and goals of care and identify opportunities to reduce inequities; and Implement strategies that encourage and support regular assessment of clinical status, LVEF, and LVOT gradient in patients with HCM, consulting with specialty colleagues as needed.
Contributor: Meghan Hurley MD Educational Pearls: Pericarditis is inflammation of the pericardial sac, which can arise from infectious or non-infectious etiologies Myocarditis is inflammation of the myocardium, which may accompany pericarditis Pericarditis clinical findings include: Diffuse concave ST elevation, classic for acute pericarditis with myocardial involvement. More common in younger male patients Elevated high-sensitivity troponin - higher levels may occur in young healthy patients Ultrasound may show pericardial effusions POCUS may be helpful in assessing left ventricular ejection fraction (LVEF) via E-point septal separation (EPSS) Elevation in EPSS correlates with decreased LVEF Treatments: Anti-inflammatories including NSAIDs and colchicine Monitor inflammation Repeat ultrasounds Risk factors in this patient's case: mRNA COVID vaccine - the risk of myocarditis from vaccination is significantly lower than that from COVID-19 infection Preceding infection References 1. Gao J, Feng L, Li Y, et al. A Systematic Review and Meta-analysis of the Association Between SARS-CoV-2 Vaccination and Myocarditis or Pericarditis. Am J Prev Med. 2023;64(2):275-284. 2. Imazio M, Gaita F, LeWinter M. Evaluation and treatment of pericarditis: A systematic review. JAMA - J Am Med Assoc. 2015;314(14):1498-1506. doi:10.1001/jama.2015.12763 3. Mckaigney CJ, Krantz MJ, La Rocque CL, Hurst ND, Buchanan MS, Kendall JL. E-point septal separation: A bedside tool for emergency physician assessment of left ventricular ejection fraction. Am J Emerg Med. 2014;32(6):493-497. doi:10.1016/j.ajem.2014.01.045 Summarized by Jorge Chalit, OMSII | Edited by Meg Joyce & Jorge Chalit, OMSII
CardioNerds (Amit Goyal and Daniel Ambider) ACHD series co-chair Dr. Daniel Clark (Vanderbilt University), cardiology FIT lead Dr. Stephanie Fuentes (Houston Methodist Hospital), and Dr. Frank Fish, a Pediatric Electrophysiologist and the Director of the Pediatric Electrophysiology (EP) Lab at Monroe Carrell Jr Children's Hospital at Vanderbilt University. He is a board certified Adult Congenital Heart Disease (ACHD) physician and has a wealth of experience performing EP procedures in adults living with congenital heart disease. Audio editing was performed by student Dr. Shivani Reddy. In this episode, we discuss key concepts and management of electrophysiologic issues that we can encounter when caring for adults with congenital heart disease. Arrythmias in adults with congenital heart disease can be intrinsic due to the defect itself or as a consequence of the interventions that they have undergone to palliate and/or repair these defects. The complex anatomy of these patients and the years of pressure and volume load make them not only exquisitely hemodynamically sensitive to arrhythmias (that may otherwise not be of much consequence to the general population) but they also make interventions (catheter ablation or device implant) complex. We therefore embark in a case-based discussion of patients with ACHD (Fontan circulation, Ebstein's anomaly and Tetralogy of Fallot) in an effort to highlight the presentation of arrythmias and the management strategy in this very important group of patients. The CardioNerds Adult Congenital Heart Disease (ACHD) series provides a comprehensive curriculum to dive deep into the labyrinthine world of congenital heart disease with the aim of empowering every CardioNerd to help improve the lives of people living with congenital heart disease. This series is multi-institutional collaborative project made possible by contributions of stellar fellow leads and expert faculty from several programs, led by series co-chairs, Dr. Josh Saef, Dr. Agnes Koczo, and Dr. Dan Clark. The CardioNerds Adult Congenital Heart Disease Series is developed in collaboration with the Adult Congenital Heart Association, The CHiP Network, and Heart University. See more CardioNerds Adult Congenital Heart Disease PageCardioNerds Episode PageCardioNerds AcademyCardionerds Healy Honor Roll CardioNerds Journal ClubSubscribe to The Heartbeat Newsletter!Check out CardioNerds SWAG!Become a CardioNerds Patron! Pearls - Electrophysiology in ACHD Patients with Fontan circulation have a high risk of developing atrial (and ventricular) arrhythmias and they are highly sensitive to the hemodynamic consequences that these arrythmias ensue. The goal of therapy then should be to achieve sinus or atrial paced rhythm. Rate control should NOT the goal. Patients with Ebstein's anomaly have high arrhythmic potential. They can have multiple accessory pathways (especially right sided) which can in turn be associated with sudden cardiac death. We should have low threshold for EPS +/- catheter ablation in patients with WPW pattern. Patients with Tetralogy of Fallot have a unique risk for SCD that warrant ICD implant apart from the standard criteria (LVEF 180 ms) and surgical repair approach. Patient's anatomy is the major consideration when implanting devices (PPM/ICD). We ought to assess for residual intracardiac shunt at the atrial level and consider closing if feasible prior to placing a device. CRT has merit in systemic LV but less so in systemic RV. Notes- Electrophysiology in ACHD What should we know about atrial arrhythmias in a Fontan patient? Intraatrial re-entrant tachycardia (IART) is slower than typical atrial flutter with atrial rates generally
The following question refers to Section 7.8 of the 2022 AHA/ACC/HFSA Guideline for the Management of Heart Failure.The question is asked by Stony Brook University Hospital medicine resident and CardioNerds Intern Dr. Chelsea Tweneboah, answered first by Mayo Clinic Cardiology Fellow and CardioNerds Academy Chief Dr. Teodora Donisan, and then by expert faculty Dr. Michelle Kittleson.The Decipher the Guidelines: 2022 AHA / ACC / HFSA Guideline for The Management of Heart Failure series was developed by the CardioNerds and created in collaboration with the American Heart Association and the Heart Failure Society of America. It was created by 30 trainees spanning college through advanced fellowship under the leadership of CardioNerds Cofounders Dr. Amit Goyal and Dr. Dan Ambinder, with mentorship from Dr. Anu Lala, Dr. Robert Mentz, and Dr. Nancy Sweitzer. We thank Dr. Judy Bezanson and Dr. Elliott Antman for tremendous guidance.Enjoy this Circulation 2022 Paths to Discovery article to learn about the CardioNerds story, mission, and values. Question #29 A 69-year-old man was referred to the cardiology clinic after being found to have a reduced left ventricular ejection fraction and left ventricular hypertrophy. For the last several months he has been experiencing progressively worsening fatigue and shortness of breath while getting to the 2nd floor in his house. He has a history of bilateral carpal tunnel syndrome and chronic low back pain. He takes no medications. On exam, his heart rate is 82 bpm, blood pressure is 86/60 mmHg, O2 saturation is 97% breathing ambient air, and BMI is 29 kg/m2. He has a regular rate and rhythm with normal S1 and S2, bibasilar pulmonary rales, and 1+ pitting edema in both legs. EKG shows normal sinus rhythm with a first-degree AV delay and low voltages. Transthoracic echocardiogram shows a moderately depressed LVEF of 35-39%, severe concentric hypertrophy with a left ventricular posterior wall thickness of 1.5 cm and strain imaging showing globally reduced longitudinal strain with apical sparring. There is also biatrial enlargement and a small pericardial effusion. A pharmacologic nuclear stress test did not reveal any perfusion defects. A gammopathy panel including SPEP, UPEP, serum and urine immunofixation studies, and serum free light chains are unrevealing. A 99mTc-Pyrophosphate scan was positive with grade 3 uptake. In addition to starting diuretics, what is the next most appropriate step for managing for this patient? A Start metoprolol succinate B Start sacubitril/valsartan C Perform genetic sequencing of the TTR gene D Perform endomyocardial biopsy Answer #29 Explanation The correct answer is C – perform genetic sequencing of the TTR gene. This patient has findings which raise suspicion for cardiac amyloidosis. There are both cardiac (low voltages on EKG and echocardiogram showing marked LVH with biatrial enlargement and small pericardial effusion as well as a characteristic strain pattern) and extra-cardiac (bilateral carpal tunnel syndrome and low back pain) features to suggest amyloidosis. The diagnosis of cardiac amyloidosis requires a high index of suspicion and most commonly occurs due to a deposition of monoclonal immunoglobulin light chains (AL-CM) or transthyretin (ATTR-CM). ATTR may cause cardiac amyloidosis as either a pathogenic variant (ATTRv) or as a wild-type protein (ATTRwt). Patients for whom there is a clinical suspicion for cardiac amyloidosis should have screening for serum and urine monoclonal light chains with serum and urine immunofixation electrophoresis and serum free light chains (Class 1, LOE B-NR). Immunofixation electrophoresis (IFE) is preferred because serum or urine plasma electrophoresis (SPEP or UPEP) are less sensitive. Together, measurement of serum IFE, urine IFE, and serum FLC is >99% sensitive for AL amyloidosis.
The following question refers to Section 7.3 of the 2022 AHA/ACC/HFSA Guideline for the Management of Heart Failure. The question is asked by Palisades Medical Center medicine resident & CardioNerds Academy Fellow Dr. Maryam Barkhordarian, answered first by Hopkins Bayview medicine resident & CardioNerds Academy Faculty Dr. Ty Sweeny, and then by expert faculty Dr. Gregg Fonarow. Dr. Fonarow is the Professor of Medicine and Interim Chief of UCLA's Division of Cardiology, Director of the Ahmanson-UCLA Cardiomyopathy Center, and Co-director of UCLA's Preventative Cardiology Program. The Decipher the Guidelines: 2022 AHA / ACC / HFSA Guideline for The Management of Heart Failure series was developed by the CardioNerds and created in collaboration with the American Heart Association and the Heart Failure Society of America. It was created by 30 trainees spanning college through advanced fellowship under the leadership of CardioNerds Cofounders Dr. Amit Goyal and Dr. Dan Ambinder, with mentorship from Dr. Anu Lala, Dr. Robert Mentz, and Dr. Nancy Sweitzer. We thank Dr. Judy Bezanson and Dr. Elliott Antman for tremendous guidance. Enjoy this Circulation 2022 Paths to Discovery article to learn about the CardioNerds story, mission, and values. Question #28 Mr. Gene D'aMeTi, a 53-year-old African American man with ischemic cardiomyopathy and heart failure with reduced ejection fraction (LVEF 30-35%), is recently admitted with acutely decompensated heart failure and acute kidney injury on chronic kidney disease stage III. His outpatient regiment includes sacubitril-valsartan 97-103mg BID, carvedilol 25mg BID, and hydralazine 50mg TID. Sacubitril-valsartan was held because of worsening renal function. Despite symptomatic improvement with diuresis, his renal function continues to decline. He is otherwise well perfused & with preservation of other end organ function. Throughout this hospitalization, he has steadily become more hypertensive with blood pressures persisting in the 170s/90s mmHg. What would be an appropriate adjustment to his medication regimen at this time? A Resume Losartan only B Start Amlodipine C Increase current Hydralazine dose D Start Isosorbide dinitrate therapy E Both C & D Answer #28 ExplanationThe correct answer is E – both increasing the current hydralazine dose (C) and starting isosorbide dinitrate therapy (D). Although ACEI/ARB therapy (choice A) has shown a mortality and morbidity benefit in HFrEF, caution should be used in patients with renal insufficiency. In this patient with ongoing decline in renal function, RAAS-inhibiting therapies (ACEi, ARB, ARNI, MRA) should be avoided. In this case, as his RAAS-I has been stopped, it would be reasonable to increase current therapies to target doses (or nearest dose tolerated), as these demonstrated both safety and efficacy in trials (Class 1, LOE A). Considering that his high dose ARNI was stopped, it is unlikely that either hydralazine or isosorbide dinitrate alone, even at maximal doses, would be sufficient to control his blood pressure (Options C and D, respectively). Interestingly, in the original study by Massie et. Al (1977), the decision was made to combine these therapies as the result was thought to be superior to either medication alone. ISDN would provide preload reduction, while Hydralazine would decrease afterload. Consequently, we do not have data looking at the individual benefit of either medication in isolation. In self-identified African Americans with NYHA class III or IV HFrEF already on optimal GDMT, the addition of hydralazine & isosorbide dinitrate is recommended to improve symptoms and reduce mortality and morbidity (Class 1, LOE A). In this case, as the patient has evidence of progressive renal disfunction, we are limited in using traditional RAAS-I, such as ACEI, ARB, or ARNI.
William H. Sauer, MD, FHRS, CCDS, is joined by guests Jorge Romero, Jr., MD, FHRS, and Esseim Sharma, MD, for a discussion of the comparison of left bundle-branch area pacing to biventricular pacing in candidates for resynchronization therapy. This observational study included patients with LVEF≤35% who underwent biventricular or left bundle area pacing for the first time for Class I or II indications at 15 international centers. The primary outcome was the composite endpoint of time to death or heart failure hospitalization. Secondary outcomes included endpoints of death, HFH, and echocardiographic changes. Following implantation, LVEF improved from 27 to 41% with left bundle area pacing compared to an increase from 27 to 37% with biventricular pacing. On multivariable regression analysis, the primary outcome was significantly reduced with left bundle area pacing compared to biventricular pacing (20.8% vs 28%). The authors conclude that left bundle area pacing improved clinical outcomes, as observed in this study. https://www.hrsonline.org/education/TheLead Host Disclosure(s): W. Sauer: Honoraria/Speaking/Consulting Fee: Biotronik, Biosense Webster, Inc., Abbott, Boston Scientific; Research (Contracted Grants for PIs Named Investigators Only): Medtronic Contributor Disclosure(s): J. Romero: Honoraria/Speaking/Consulting Fee: Sanofi, Boston Scientific, AtriCure, Inc.; Research (Contracted Grants for PIs Named Investigators Only): Biosense Webster, Inc. E. Sharma: No relevant financial relationships with ineligible companies to disclose.
The following question refers to Section 4.3 of the 2022 AHA/ACC/HFSA Guideline for the Management of Heart Failure.The question is asked by Texas Tech University medical student and CardioNerds Academy Intern Dr. Adriana Mares, answered first by Rochester General Hospital cardiology fellow and Director of CardioNerds Journal Club Dr. Devesh Rai, and then by expert faculty Dr. Eldrin Lewis.Dr. Lewis is an Advanced Heart Failure and Transplant Cardiologist, Professor of Medicine and Chief of the Division of Cardiovascular Medicine at Stanford University. The Decipher the Guidelines: 2022 AHA / ACC / HFSA Guideline for The Management of Heart Failure series was developed by the CardioNerds and created in collaboration with the American Heart Association and the Heart Failure Society of America. It was created by 30 trainees spanning college through advanced fellowship under the leadership of CardioNerds Cofounders Dr. Amit Goyal and Dr. Dan Ambinder, with mentorship from Dr. Anu Lala, Dr. Robert Mentz, and Dr. Nancy Sweitzer. We thank Dr. Judy Bezanson and Dr. Elliott Antman for tremendous guidance.Enjoy this Circulation 2022 Paths to Discovery article to learn about the CardioNerds story, mission, and values. Question #26 A 45-year-old man presents to cardiology clinic to establish care. He has had several months of progressive dyspnea on exertion while playing basketball. He also reports intermittent palpitations for the last month. Two weeks ago, he passed out while playing and attributed this to exertion and dehydration. He denies smoking and alcohol intake. Family history is significant for sudden cardiac death in his father at the age of 50 years. Autopsy has shown a thick heart, but he is unaware of the exact diagnosis. He has two children, ages 12 and 15 years old, who are healthy. Vitals signs are blood pressure of 124/84 mmHg, heart rate of 70 bpm, and normal respiratory rate. On auscultation, a systolic murmur is present at the left lower sternal border. A 12-lead ECG showed normal sinus rhythm with signs of LVH and associated repolarization abnormalities. Echocardiography reveals normal LV chamber volume, preserved LVEF, asymmetric septal hypertrophy with wall thickness up to 16mm, systolic anterior motion of the anterior mitral valve leaflet with 2+ eccentric posteriorly directed MR, and resting LVOT gradient of 30mmHg which increases to 60mmHg on Valsalva. You discuss your concern for an inherited cardiomyopathy, namely hypertrophic cardiomyopathy. In addition to medical management of his symptoms and referral to electrophysiology for ICD evaluation, which of the following is appropriate at this time? A Order blood work for genetic testing B Referral for genetic counseling C Cardiac MRI D Coronary angiogram E All of the above Answer #26 Explanation The correct answer is B – referral for genetic counseling. Several factors on clinical evaluation may indicate a possible underlying genetic cardiomyopathy. Clues may be found in: · Cardiac morphology – marked LV hypertrophy, LV noncompaction, RV thinning or fatty replacement on imaging or biopsy · 12-lead ECG – abnormal high or low voltage or conduction, and repolarization, altered RV forces · Presence of arrhythmias – frequent NSVT or very frequent PVCs, sustained VT or VF, early onset AF, early onset conduction disease · Extracardiac features – skeletal myopathy, neuropathy, cutaneous stigmata, and other possible manifestations of specific syndromes In select patients with nonischemic cardiomyopathy, referral for genetic counseling and testing is reasonable to identify conditions that could guide treatment for patients and family members (Class 2a, LOE B-NR). In first-degree relatives of selected patients with genetic or inherited cardiomyopathies, genetic screening and counseling are recommended to ...
The following question refers to Sections 6.1 and 7.3 of the 2022 AHA/ACC/HFSA Guideline for the Management of Heart Failure. The question is asked by Keck School of Medicine USC medical student & former CardioNerds Intern Hirsh Elhence, answered first by Greater Baltimore Medical Center medicine resident and CardioNerds Academy Fellow Dr. Alaa Diab, and then by expert faculty Dr. Mark Drazner. Dr. Drazner is an advanced heart failure and transplant cardiologist, Professor of Medicine, and Clinical Chief of Cardiology at UT Southwestern. He is the past President of the Heart Failure Society of America. The Decipher the Guidelines: 2022 AHA / ACC / HFSA Guideline for The Management of Heart Failure series was developed by the CardioNerds and created in collaboration with the American Heart Association and the Heart Failure Society of America. It was created by 30 trainees spanning college through advanced fellowship under the leadership of CardioNerds Cofounders Dr. Amit Goyal and Dr. Dan Ambinder, with mentorship from Dr. Anu Lala, Dr. Robert Mentz, and Dr. Nancy Sweitzer. We thank Dr. Judy Bezanson and Dr. Elliott Antman for tremendous guidance. Enjoy this Circulation 2022 Paths to Discovery article to learn about the CardioNerds story, mission, and values. Question #25 A 50-year-old man with a history of type 2 diabetes mellitus, persistent atrial fibrillation, coronary artery disease with prior remote percutaneous coronary intervention, and ischemic cardiomyopathy with HFrEF (LVEF 38%) presents to your outpatient clinic. He denies dyspnea on exertion, orthopnea, bendopnea, paroxysmal nocturnal dyspnea, or peripheral edema. His heart rate is irregularly irregular at 112 beats per minute and blood pressure is 112/67 mmHg. Routine laboratory studies reveal a hemoglobin A1c of 7.7%. Which of the following medications should not be used to control this patient's comorbidities? A Metoprolol succinate B Verapamil C Dapagliflozin D Pioglitizone E Both B and D Answer #25 Explanation The correct answer is E – both verapamil and pioglitazone should be avoided here. Both verapamil and pioglitizone are associated with harm in patients with LVEF < 50% (Class 3: Harm). Verapamil and diltiazem are non-dihydropyridine calcium channel blockers. These medications can cause negative inotropic effects through inhibition of calcium influx and may be harmful in this patient population. Pioglitizone belongs to a class of diabetic medications known as the thiazolidinediones. This class of medications may increase the risk of fluid retention, heart failure, and hospitalization in patients with LVEF of less than 50%. Metoprolol succinate, and other evidence-based beta blockers, have a Class 1 recommendation for patients with reduced ejection fraction ≤ 40% to prevent symptomatic heart failure and reduce mortality. It may additionally help with rate control in this patient with atrial fibrillation and rapid ventricular response. SGLT2 inhibitors including dapagliflozin have a Class I recommendation for patients with symptomatic chronic HFrEF to reduce hospitalization for HF and cardiovascular mortality, irrespective of the presence of type 2 diabetes (Class 1, LOE A). They also have a Class I recommendation in patients with type 2 diabetes and either established CVD or at high cardiovascular risk to prevent hospitalization for HF (Class 1, LOE A). Our patient has asymptomatic, or pre-HF (Stage B) heart failure with poorly controlled diabetes, and so use of an SGLT2 inhibitor would be appropriate. Main Takeaway Non-dihydropyridine calcium channel blockers and thiozolidinediones both have Class 3 recommendations for harm in patients with reduced LV systolic dysfunction. Guideline Loc. Section 6.1 and 7.3 Decipher the Guidelines: 2022 Heart Failure Guidelines PageCardioNerds Episode PageCardioNerds Academ...
The following question refers to Sections 10.2 of the 2022 AHA/ACC/HFSA Guideline for the Management of Heart Failure. The question is asked by Western Michigan University medical student and CardioNerds Intern Shivani Reddy, answered first by Mayo Clinic Cardiology Fellow and CardioNerds Academy House Faculty Leader Dr. Dinu Balanescu, and then by expert faculty Dr. Ileana Pina. Dr. Pina is Professor of Medicine and Quality Officer for the Cardiovascular Line at Thomas Jefferson University, Clinical Professor at Central Michigan University, and Adjunct Professor of Biostats and Epidemiology at Case Western University. She serves as Senior Fellow and Medical Officer at the Food and Drug Administration's Center for Devices and Radiological Health. The Decipher the Guidelines: 2022 AHA / ACC / HFSA Guideline for The Management of Heart Failure series was developed by the CardioNerds and created in collaboration with the American Heart Association and the Heart Failure Society of America. It was created by 30 trainees spanning college through advanced fellowship under the leadership of CardioNerds Cofounders Dr. Amit Goyal and Dr. Dan Ambinder, with mentorship from Dr. Anu Lala, Dr. Robert Mentz, and Dr. Nancy Sweitzer. We thank Dr. Judy Bezanson and Dr. Elliott Antman for tremendous guidance. Enjoy this Circulation 2022 Paths to Discovery article to learn about the CardioNerds story, mission, and values. Question #24 Mr. E. Regular is a 61-year-old man with a history of HFrEF due to non-ischemic cardiomyopathy (latest LVEF 40% after >3 months of optimized GDMT) and persistent atrial fibrillation. He has no other medical history. He has been on metoprolol and apixaban and has also undergone multiple electrical cardioversions and catheter ablations for atrial fibrillation but remains symptomatic with poorly controlled rates. His blood pressure is 105/65 mm Hg. HbA1c is 5.4%. Which of the following is a reasonable next step in the management of his atrial fibrillation? A Anti-arrhythmic drug therapy with amiodarone. Stop apixaban. B Repeat catheter ablation for atrial fibrillation. Stop apixaban. C AV nodal ablation and RV pacing. Shared decision-making regarding anticoagulation. D AV nodal ablation and CRT device. Shared decision-making regarding anticoagulation. Answer #24 Explanation The correct answer is D – AV nodal ablation and CRT device along with shared decision-making regarding anticoagulation.” Maintaining sinus rhythm and atrial-ventricular synchrony is helpful in patients with heart failure given the hemodynamic benefits of atrial systole for diastolic filling and having a regularized rhythm. Recent randomized controlled trials suggest that catheter-based rhythm control strategies are superior to rate control and chemical rhythm control strategies with regards to outcomes in atrial fibrillation. For patients with heart failure and symptoms caused by atrial fibrillation, ablation is reasonable to improve symptoms and quality of life (Class 2a, LOE B-R). However, Mr. Regular has already had multiple failed attempts at ablations (option B). For patients with AF and LVEF ≤50%, if a rhythm control strategy fails or is not desired, and ventricular rates remain rapid despite medical therapy, atrioventricular nodal ablation with implantation of a CRT device is reasonable (Class 2a, LOE B-R). The PAVE and BLOCK-HF trials suggested improved outcomes with CRT devices in these patients. RV pacing following AV nodal ablation has also been shown to improve outcomes in patients with atrial fibrillation refractory to other rhythm control strategies. In patients with EF >50%, there is no evidence to suggest that CRT is more beneficial compared to RV-only pacing. However, RV pacing may produce ventricular dyssynchrony and when compared to CRT in those with reduced EF (≤ 50%),
The following question refers to Section 9.3 of the 2022 AHA/ACC/HFSA Guideline for the Management of Heart Failure.The question is asked by Keck School of Medicine USC medical student & CardioNerds Intern Hirsh Elhence, answered first by Cedars Sinai medicine resident, soon to be Vanderbilt Cardiology Fellow, and CardioNerds Academy Faculty Dr. Breanna Hansen, and then by expert faculty Dr. Anu Lala.Dr. Lala is an advanced heart failure and transplant cardiologist, associate professor of medicine and population health science and policy, Director of Heart Failure Research, and Program Director for the Advanced Heart Failure and Transplant fellowship training program at Mount Sinai. Dr. Lala is Deputy Editor for the Journal of Cardiac Failure. Dr. Lala has been a champion and role model for CardioNerds. She has been a PI mentor for the CardioNerds Clinical Trials Network and continues to serve in the program's leadership. She is also a faculty mentor for this very 2022 heart failure decipher the guidelines series.The Decipher the Guidelines: 2022 AHA / ACC / HFSA Guideline for The Management of Heart Failure series was developed by the CardioNerds and created in collaboration with the American Heart Association and the Heart Failure Society of America. It was created by 30 trainees spanning college through advanced fellowship under the leadership of CardioNerds Cofounders Dr. Amit Goyal and Dr. Dan Ambinder, with mentorship from Dr. Anu Lala, Dr. Robert Mentz, and Dr. Nancy Sweitzer. We thank Dr. Judy Bezanson and Dr. Elliott Antman for tremendous guidance.Enjoy this Circulation 2022 Paths to Discovery article to learn about the CardioNerds story, mission, and values. Question #23 Mrs. Hart is a 63-year-old woman with a history of non-ischemic cardiomyopathy and heart failure with reduced ejection fraction (LVEF 20-25%) presenting with 5 days of worsening dyspnea and orthopnea. At home, she takes carvedilol 12.5mg BID, sacubitril-valsartan 24-46mg BID, empagliflozin 10mg daily, and furosemide 40mg daily. On admission, her exam revealed a blood pressure of 111/79 mmHg, HR 80 bpm, and SpO2 94%. Her cardiovascular exam was significant for a regular rate and rhythm with an audible S3, JVD to 13 cm H2O, bilateral lower extremity pitting edema with warm extremities and 2+ pulses throughout. What initial dose of diuretics would you give her? A Continue home Furosemide 40 mg PO B Start Metolazone 5 mg PO C Start Lasix 100 mg IV D Start Spironolactone Answer #23 Explanation The correct answer is C – start Furosemide 100 mg IV. This is the most appropriate choice because patients with HF admitted with evidence of significant fluid overload should be promptly treated with intravenous loop diuretics to improve symptoms and reduce morbidity (Class 1, LOE B-NR). Intravenous loop diuretic therapy provides the most rapid and effective treatment for signs and symptoms of congestion. Titration of diuretics has been described in multiple recent trials of patients hospitalized with HF, often initiated with at least 2 times the daily home diuretic dose (mg to mg) administered intravenously. Titration to achieve effective diuresis may require doubling of initial doses, adding a thiazide diuretic, or adding an MRA that has diuretic effects in addition to its cardiovascular benefits. Choice A is incorrect as continuing oral loop diuretics is not recommended for acute decongestion. Moreover, Ms. Hart has become congested despite her home, oral diuretic regimen. Choice B and D are incorrect as starting a thiazide diuretic or a mineralocorticoid receptor antagonist are not first-line therapy for acute HF. Rather, in patients hospitalized with HF when diuresis is inadequate to relieve symptoms and signs of congestion, it is reasonable to intensify the diuretic regimen using either: a.
The following question refers to Section 8.3 of the 2022 AHA/ACC/HFSA Guideline for the Management of Heart Failure. The question is asked by Western Michigan University medical student & CardioNerds Intern Shivani Reddy, answered first by University of Southern California cardiology fellow and CardioNerds FIT Trialist Dr. Michael Francke, and then by expert faculty Dr. Prateeti Khazanie. Dr. Khazanie is an associate professor and advanced heart failure and transplant Cardiologist at the University of Colorado. Dr. Khazanie is an author on the 2022 ACC/AHA/HFSA HF Guidelines, the 2021 HFSA Universal Definition of Heart Failure, and multiple scientific statements. The Decipher the Guidelines: 2022 AHA / ACC / HFSA Guideline for The Management of Heart Failure series was developed by the CardioNerds and created in collaboration with the American Heart Association and the Heart Failure Society of America. It was created by 30 trainees spanning college through advanced fellowship under the leadership of CardioNerds Cofounders Dr. Amit Goyal and Dr. Dan Ambinder, with mentorship from Dr. Anu Lala, Dr. Robert Mentz, and Dr. Nancy Sweitzer. We thank Dr. Judy Bezanson and Dr. Elliott Antman for tremendous guidance. Enjoy this Circulation 2022 Paths to Discovery article to learn about the CardioNerds story, mission, and values. Clinical Trials Talks Question #22 You are taking care of a 34-year-old man with chronic systolic heart failure from NICM with LVEF 20% s/p CRT-D. The patient was admitted 1 week prior with acute decompensated heart failure. Despite intravenous diuretics the patient developed acute kidney injury, and ultimately placed on intravenous inotropes on which he now seems dependent. He has been following up with an advanced heart failure specialist as an outpatient and has been undergoing evaluation for heart transplantation, which was subsequently completed in the hospital. His exam is notable for an elevated JVP, a III/VI holosystolic murmur, and warm extremities with bilateral 1+ edema. His most recent TTE shows LVEF 20%, moderate MR, moderate-severe TR and estimated RVSP 34 mmHg. His most recent laboratory data shows Na 131 mmol/L, Cr 1.2 mg/dL, and lactate 1.6 mmol/L. Pulmonary artery catheter shows RA 7 mmHg, PA 36/15 mmHg, PCWP 12 mmHg, CI 2.4 L/min/m2 and SVR 1150 dynes*sec/cm5. The patient was presented at transplant selection committee and approved for listing for orthotopic heart transplant. What is the most appropriate next step in the management of this patient? A Refer patient for transcatheter edge-to-edge repair for MR B Continue IV inotropes as a bridge-to-transplant C Refer patient for tricuspid valve replacement D Initiate 1.5L fluid restriction Answer #22 Explanation The correct answer is B – continue IV inotropes as a bridge-to-transplant. Positive inotropic agents may improve hemodynamic status, but have not been shown to improve survival in patients with HF. These agents may help HF patients who are refractory to other therapies and are suffering consequences from end-organ-hypoperfusion. Our patient is admitted with worsening advanced heart failure requiring intravenous inotropic support. He has been appropriately evaluated and approved for heart transplant. He has demonstrated the requirement of continuous inotropic support to maintain perfusion. In patients such as this with advanced (stage D) HF refractory to GDMT and device therapy who are eligible for and awaiting MCS or cardiac transplantation, continuous intravenous inotropic support is reasonable as “bridge therapy” (Class 2a, LOE B-NR). Continuous IV inotropes also have a Class 2b indication (LOE B-NR) in select patients with stage D HF despite optimal GDMT and device therapy who are ineligible for either MCS or cardiac transplantation, as palliative therapy for symptom control and improvement in functio...
The following question refers to Section 7.6 of the 2022 AHA/ACC/HFSA Guideline for the Management of Heart Failure.The question is asked by premedical student and CardioNerds Intern Pacey Wetstein, answered first by Mayo Clinic Cardiology Fellow and CardioNerds Academy Chief Dr. Teodora Donisan, and then by expert faculty Dr. Nancy Sweitzer.Dr. Sweitzer is Professor of Medicine, Vice Chair of Clinical Research for the Department of Medicine, and Director of Clinical Research for the Division of Cardiology at Washington University School of Medicine. She is the editor-in-chief of Circulation: Heart Failure. Dr. Sweitzer is a faculty mentor for this Decipher the HF Guidelines series.The Decipher the Guidelines: 2022 AHA / ACC / HFSA Guideline for The Management of Heart Failure series was developed by the CardioNerds and created in collaboration with the American Heart Association and the Heart Failure Society of America. It was created by 30 trainees spanning college through advanced fellowship under the leadership of CardioNerds Cofounders Dr. Amit Goyal and Dr. Dan Ambinder, with mentorship from Dr. Anu Lala, Dr. Robert Mentz, and Dr. Nancy Sweitzer. We thank Dr. Judy Bezanson and Dr. Elliott Antman for tremendous guidance.Enjoy this Circulation 2022 Paths to Discovery article to learn about the CardioNerds story, mission, and values. /*! elementor - v3.13.3 - 28-05-2023 */ .elementor-heading-title{padding:0;margin:0;line-height:1}.elementor-widget-heading .elementor-heading-title[class*=elementor-size-]>a{color:inherit;font-size:inherit;line-height:inherit}.elementor-widget-heading .elementor-heading-title.elementor-size-small{font-size:15px}.elementor-widget-heading .elementor-heading-title.elementor-size-medium{font-size:19px}.elementor-widget-heading .elementor-heading-title.elementor-size-large{font-size:29px}.elementor-widget-heading .elementor-heading-title.elementor-size-xl{font-size:39px}.elementor-widget-heading .elementor-heading-title.elementor-size-xxl{font-size:59px}Clinical Trials Talks Question #21 Ms. Betty Blocker is a 60-year-old woman with a history of alcohol-related dilated cardiomyopathy who presents for follow up. She has been working hard to improve her health and is glad to report that she has just reached her 5-year sobriety milestone. Her current medications include metoprolol succinate 100mg daily, sacubitril-valsartan 97-103mg BID, spironolactone 25mg daily, and empagliflozin 10mg daily. She is asymptomatic at rest and up to moderate exercise, including chasing her grandchildren around the yard. A recent transthoracic echocardiogram shows recovered LVEF from previously 35% now to 60%. Ms. Blocker does not love taking so many medications and asks about discontinuing her metoprolol. Which of the following is the most appropriate response to Ms. Blocker's request? A Since the patient is asymptomatic, metoprolol can be stopped without risk B Stopping metoprolol increases this patient's risk of worsening cardiomyopathy regardless of current LVEF or symptoms C Because the LVEF is now >50%, the patient is now classified as having HFpEF and beta-blockade is no longer indicated; metoprolol can be safely discontinued D Metoprolol should be continued, but it is safe to discontinue either ARNi or spironolactone Answer #21 Explanation The correct answer is D – continue current therapy. The patient described above was initially diagnosed with HFrEF and experienced significant symptomatic improvement with GDMT, so she now has heart failure with improved ejection fraction (HFimpEF). In patients with HFimpEF after treatment, GDMT should be continued to prevent relapse of HF and LV dysfunction, even in patients who may become asymptomatic (Class 1, LOE B-R). Although symptoms, functional capacity, LVEF and reverse remodeling can improve with GDMT,
The following question refers to Sections 7.3.2, 7.3.8, and 7.6.2 of the 2022 AHA/ACC/HFSA Guideline for the Management of Heart Failure. The question is asked by Palisades Medical Center medicine resident & CardioNerds Intern Dr. Maryam Barkhordarian, answered first by Hopkins Bayview medicine resident & CardioNerds Academy Fellow Dr. Ty Sweeny, and then by expert faculty Dr. Robert Mentz. Dr. Mentz is associate professor of medicine and section chief for Heart Failure at Duke University, a clinical researcher at the Duke Clinical Research Institute, and editor-in-chief of the Journal of Cardiac Failure. Dr. Mentz is a mentor for the CardioNerds Clinical Trials Network as lead principal investigator for PARAGLIDE-HF and is a series mentor for this very Decipher the Guidelines Series. For these reasons and many more, he was awarded the Master CardioNerd Award during ACC22. The Decipher the Guidelines: 2022 AHA / ACC / HFSA Guideline for The Management of Heart Failure series was developed by the CardioNerds and created in collaboration with the American Heart Association and the Heart Failure Society of America. It was created by 30 trainees spanning college through advanced fellowship under the leadership of CardioNerds Cofounders Dr. Amit Goyal and Dr. Dan Ambinder, with mentorship from Dr. Anu Lala, Dr. Robert Mentz, and Dr. Nancy Sweitzer. We thank Dr. Judy Bezanson and Dr. Elliott Antman for tremendous guidance. Enjoy this Circulation 2022 Paths to Discovery article to learn about the CardioNerds story, mission, and values. Question #20 Ms. Betty Blocker is a 60-year-old woman with a history of alcohol-related dilated cardiomyopathy who presents for follow up. She has been working hard to improve her health and is glad to report that she has just reached her 5-year sobriety milestone. Her current medications include metoprolol succinate 100mg daily, sacubitril-valsartan 97-103mg BID, spironolactone 25mg daily, and empagliflozin 10mg daily. She is asymptomatic at rest and up to moderate exercise, including chasing her grandchildren around the yard. A recent transthoracic echocardiogram shows recovered LVEF from previously 35% now to 60%. Ms. Blocker does not love taking so many medications and asks about discontinuing her metoprolol. Which of the following is the most appropriate response to Ms. Blocker's request? A Since the patient is asymptomatic, metoprolol can be stopped without risk B Stopping metoprolol increases this patient's risk of worsening cardiomyopathy regardless of current LVEF or symptoms C Because the LVEF is now >50%, the patient is now classified as having HFpEF and beta-blockade is no longer indicated; metoprolol can be safely discontinued D Metoprolol should be continued, but it is safe to discontinue either ARNi or spironolactone Answer #20 Explanation The correct answer is B – stopping metoprolol would increase her risk of worsening cardiomyopathy. Heart failure tends to be a chronically sympathetic state. The use of beta-blockers (specifically bisoprolol, metoprolol succinate, and carvedilol) targets this excess adrenergic output and has been shown to reduce the risk of death in patients with HFrEF. Beyond their mortality benefit, beta-blockers can improve LVEF, lessen the symptoms of HF, and improve clinical status. Therefore, in patients with HFrEF, with current or previous symptoms, use of 1 of the 3 beta blockers proven to reduce mortality (e.g., bisoprolol, carvedilol, sustained-release metoprolol succinate) is recommended to reduce mortality and hospitalizations (Class 1, LOE A). Beta-blockers in this setting provide a high economic value. Table 14 of the guidelines provides recommendations for target doses for GDMT medications. Specifically for beta blockers, those targets are 25-50mg twice daily for carvedilol (or 80mg once daily for the continuous release formulation), 200mg once daily for metoprolol succinate,
The following question refers to Sections 11.3 of the 2022 AHA/ACC/HFSA Guideline for the Management of Heart Failure. The question is asked by Western Michigan University medical student and CardioNerds Intern Shivani Reddy, answered first by Johns Hopkins Osler internal medicine resident and CardioNerds Academy Fellow Dr. Justin Brilliant, and then by expert faculty Dr. Harriette Van Spall. Dr. Van Spall is Associate Professor of Medicine, cardiologist, and Director of E-Health at McMaster University. Dr Van Spall is a Canadian Institutes of Health Research-funded clinical trialist and researcher with a focus on heart failure, health services, and health disparities. The Decipher the Guidelines: 2022 AHA / ACC / HFSA Guideline for The Management of Heart Failure series was developed by the CardioNerds and created in collaboration with the American Heart Association and the Heart Failure Society of America. It was created by 30 trainees spanning college through advanced fellowship under the leadership of CardioNerds Cofounders Dr. Amit Goyal and Dr. Dan Ambinder, with mentorship from Dr. Anu Lala, Dr. Robert Mentz, and Dr. Nancy Sweitzer. We thank Dr. Judy Bezanson and Dr. Elliott Antman for tremendous guidance. Enjoy this Circulation 2022 Paths to Discovery article to learn about the CardioNerds story, mission, and values. Question #16 Ms. Augustin is a 33 y/o G1P1 woman from Haiti who seeks counseling regarding family planning as she and her husband dream of a second child. Her 1st pregnancy 12 months ago was complicated by pre-eclampsia and peripartum cardiomyopathy (LVEF 35%). Thankfully she delivered a healthy baby via C-section. She has no other past medical history and is currently on losartan 25 mg daily and metoprolol succinate 200 mg daily. She has been asymptomatic. Which of the following statements is recommended to medically optimize Ms. Augustin prior to her 2nd pregnancy? A No medical optimization or preconception planning is needed as her 1st pregnancy resulted in a healthy infant. B Discontinue losartan and metoprolol with no other needed pregnancy planning C Change her medication regimen, consider repeat TTE, and provide patient-centered counseling regarding risk of a future pregnancy D Continue losartan and metoprolol and advise against repeat pregnancy Answer #16 Explanation The correct answer is C – change her medication regimen, consider repeat TTE, and provide patient-centered counseling regarding risk of a future pregnancy. Heart failure may complicate pregnancy either secondary to an existing pre-pregnancy cardiomyopathy or as a result of peripartum cardiomyopathy. In women with history of heart failure or cardiomyopathy, including previous peripartum cardiomyopathy, patient-centered counseling regarding contraception and the risks of cardiovascular deterioration during pregnancy should be provided (Class I, LOE C-LD) Peripartum cardiomyopathy (PPCM) is defined as systolic dysfunction, typically LVEF < 45%, often with LV dilation, occurring in late pregnancy or early postpartum with no other identifiable etiology. PPCM occurs worldwide, with the highest incidences in Haiti, Nigeria, and South Africa. Other clinical risk factors include maternal age > 30 years, African ancestry, multiparity, multigestation, preeclampsia/eclampsia, anemia, diabetes, obesity, and prolonged tocolysis. The pathogenesis of peripartum cardiomyopathy is complex and it is likely a multifactorial process. The combination of hemodynamic changes of pregnancy, inflammation of the myocardium, hormonal changes, genetic factors, and an autoimmune response have all been proposed as possible mechanisms and these may certainly be interrelated. While pregnancy is generally well-tolerated in women with cardiomyopathy and NYHA class I status pre-pregnancy, clinical deterioration can occur and so counseling a...
The following question refers to Section 10.1 of the 2022 AHA/ACC/HFSA Guideline for the Management of Heart Failure. The question is asked by Western Michigan University medical student and CardioNerds Intern Shivani Reddy, answered first by Boston University cardiology fellow and CardioNerds Ambassador Dr. Alex Pipilas, and then by expert faculty Dr. Ileana Pina.Dr. Pina is Professor of Medicine and Quality Officer for the Cardiovascular Line at Thomas Jefferson University, Clinical Professor at Central Michigan University, and Adjunct Professor of Biostats and Epidemiology at Case Western University. She serves as Senior Fellow and Medical Officer at the Food and Drug Administration's Center for Devices and Radiological Health.The Decipher the Guidelines: 2022 AHA / ACC / HFSA Guideline for The Management of Heart Failure series was developed by the CardioNerds and created in collaboration with the American Heart Association and the Heart Failure Society of America. It was created by 30 trainees spanning college through advanced fellowship under the leadership of CardioNerds Cofounders Dr. Amit Goyal and Dr. Dan Ambinder, with mentorship from Dr. Anu Lala, Dr. Robert Mentz, and Dr. Nancy Sweitzer. We thank Dr. Judy Bezanson and Dr. Elliott Antman for tremendous guidance.Enjoy this Circulation 2022 Paths to Discovery article to learn about the CardioNerds story, mission, and values. Question #15 Mrs. Framingham is a 65-year-old woman who presents to her cardiologist's office for stable angina and worsening dyspnea on minimal exertion. She has a history of non-insulin dependent type 2 diabetes mellitus and hypertension. She is taking metformin, linagliptin, lisinopril, and amlodipine. Blood pressure is 119/70 mmHg. Labs are notable for a hemoglobin of 14.2 mg/dL, iron of 18 mcg/dL, ferritin 150 ug/L, transferrin saturation 15%, and normal creatine kinase. An echocardiogram shows reduced left ventricular ejection fraction of 25%. Coronary angiography shows obstructive lesions involving the proximal left anterior descending, left circumflex, and right coronary arteries. In addition to optimizing GDMT, which of the following are recommendations for changes in management? A Anticoagulation, percutaneous revascularization, and IV iron B A change in her diabetic regimen, percutaneous revascularization, and PO iron C A change in her diabetic regimen, surgical revascularization, and IV iron D A change in her diabetic regimen, medical treatment alone for CAD, and PO iron E Anticoagulation and surgical revascularization Answer #15 Explanation The correct answer is C – a change in her diabetic regimen, surgical treatment and IV iron. Multimorbidity is common in patients with heart failure. More than 85% of patients with HF also have at least 2 additional chronic conditions, of which the most common are hypertension, ischemic heart disease, diabetes, anemia, chronic kidney disease, morbid obesity, frailty, and malnutrition. These conditions can markedly impact patients' tolerance to GDMT and can inform prognosis. Not only was Mrs. F found with HFrEF (most likely due to ischemic cardiomyopathy), but she also suffers from severe multi-vessel coronary artery disease, hypertension, and non-insulin dependent type 2 diabetes mellitus. In addition to starting optimized GDMT for HF, specific comorbidities in the heart failure patient warrant specific treatment strategies. Mrs. Framingham would benefit from a change in her diabetic regimen, namely switching from linagliptin to an SGLT2 inhibitor (e.g., empagliflozin, dapagliflozin). In patients with HF and type 2 diabetes, the use of SGLT2i is recommended for the management of hyperglycemia and to reduce HF related morbidity and mortality (Class 1, LOE A). Furthermore, as she has diabetes, symptomatic severe multi-vessel CAD, and LVEF≤35%,
It's another session of CardioNerds Rounds! In these rounds, Dr. Jenna Skowronski (Chief FIT at University of Pittsburgh) and Dr. Natalie Stokes (Formerly FIT at University of Pittsburgh and now General Cardiology Faculty at University of Pittsburgh) join transformational leader, educator and researcher, Dr. Mary Norine Walsh (Director of Heart Failure and Transplantation at Ascension St. Vincent Heart Center and Program Director of AHFT at St. Vincent) to discuss cardio-obstetrics and heart failure cases. Amongst her many accomplishments, Dr. Walsh is past president of the American College of Cardiology, Deputy Editor of JACC Case Reports, and a preeminent voice and thought leader in women's cardiovascular health. Audio editing by CardioNerds academy intern, Pace Wetstein. This episode is supported with unrestricted funding from Zoll LifeVest. A special thank you to Mitzy Applegate and Ivan Chevere for their production skills that help make CardioNerds Rounds such an amazing success. All CardioNerds content is planned, produced, and reviewed solely by CardioNerds. Case details are altered to protect patient health information. CardioNerds Rounds is co-chaired by Dr. Karan Desai and Dr. Natalie Stokes. CardioNerds Rounds PageCardioNerds Episode PageCardioNerds AcademyCardionerds Healy Honor Roll CardioNerds Journal ClubSubscribe to The Heartbeat Newsletter!Check out CardioNerds SWAG!Become a CardioNerds Patron! Show notes - Cardio-Obstetrics and Heart Failure Case 1 Synopsis: A woman in her earlier 30s, G1P1, with a history significant for peripartum cardiomyopathy presents to clinic for pre-conception counseling. Her prior pregnancy was in her late 20s with an uneventful pre-natal course and a spontaneous vaginal delivery at 37w2d. Two weeks after delivery, she experienced symptoms of heart failure and was found to have a new diagnosis of HFrEF. At that time TTE showed LVEF 30-35%, LVIDd 5.1cm (top normal size), diffuse hypokinesis. At that time, she was diuresed and discharged on metoprolol succinate 25mg po daily and furosemide 20mg po daily. She had one follow up visit 6 months postpartum and the furosemide was discontinued. Today in your office, she has NYHA Class I symptoms with no signs of symptoms of congestion. She walks daily and does vigorous exercise 1-2 times per week, while remaining on metoprolol. Repeat TTE with LVEF 45-50% and similar LV size. She would like to have another child and was referred to you for counseling. Case 1 Rounding Pearls: Dr. Walsh discussed extensively the importance of full GDMT in this patient who was initially undertreated with only a beta blocker. If patients are breastfeeding, clinicians should consider the addition of ACE-Inhibitor and Spironolactone. Otherwise, if not breastfeeding, they should receive maximally tolerated doses of full GDMT. For more details on medical therapy for Heart Failure during pregnancy and after, refer to this previous CardioNerds Episode with Dr. Julie Damp. Patients with peripartum cardiomyopathy are at highest risk of worsening LV systolic function when they have persistent LV systolic dysfunction from their initial diagnosis. In this circumstance, shared decision making is paramount. These patients should receive counseling on contraception and risk of pregnancy on worsening LV function, death, & fetal demise. In addition, counseling includes discussing with patients limited options in some states for complete, comprehensive reproductive care, including pregnancy termination. If patients with prior peripartum cardiomyopathy do become pregnant, a team-based approach including cardiologists, maternal fetal medicine, and obstetrics (amongst other team members) is essential to determine care & delivery timing/method. These patients should also be examined for signs of decompensation throughout the pregnancy, including rales, S3 or a reported history of PND.
The following question refers to Section 9.1 of the 2022 AHA/ACC/HFSA Guideline for the Management of Heart Failure. The question is asked by Keck School of Medicine USC medical student & CardioNerds Intern Hirsh Elhence, answered first by Duke University cardiology fellow and CardioNerds FIT Ambassador Dr. Aman Kansal, and then by expert faculty Dr. Anu Lala. Dr. Lala is an advanced heart failure and transplant cardiologist, associate professor of medicine and population health science and policy, Director of Heart Failure Research, and Program Director for the Advanced Heart Failure and Transplant fellowship training program at Mount Sinai. Dr. Lala is deputy editor for the Journal of Cardiac Failure. Dr. Lala has been a champion and role model for CardioNerds. She has been a PI mentor for the CardioNerds Clinical Trials Network and continues to serve in the program's leadership. She is also a faculty mentor for this very 2022 heart failure decipher the guidelines series. The Decipher the Guidelines: 2022 AHA / ACC / HFSA Guideline for The Management of Heart Failure series was developed by the CardioNerds and created in collaboration with the American Heart Association and the Heart Failure Society of America. It was created by 30 trainees spanning college through advanced fellowship under the leadership of CardioNerds Cofounders Dr. Amit Goyal and Dr. Dan Ambinder, with mentorship from Dr. Anu Lala, Dr. Robert Mentz, and Dr. Nancy Sweitzer. We thank Dr. Judy Bezanson and Dr. Elliott Antman for tremendous guidance. Enjoy this Circulation 2022 Paths to Discovery article to learn about the CardioNerds story, mission, and values. Question #13 Mrs. Hart is a 63-year-old woman with a history of non-ischemic cardiomyopathy and heart failure with reduced ejection fraction (LVEF 20-25%) presenting with 5 days of worsening dyspnea and orthopnea. She takes carvedilol 12.5mg BID, sacubitril-valsartan 24-46mg BID, empagliflozin 10mg daily, and furosemide 40mg daily and reports that she has been able to take all her medications. What is the initial management for Mrs. H? A Assess her degree of congestion and hypoperfusion B Search for precipitating factors C Evaluate her overall trajectory D All of the above E None of the above Answer #13 Explanation The correct answer is D – all of the above. Choice A is correct because in patients hospitalized with heart failure, the severity of congestion and adequacy of perfusion should be assessed to guide triage and initial therapy (Class 1, LOE C-LD). Congestion can be assessed by using the clinical exam to gauge right and left-sided filling pressures (e.g., elevated JVP, S3, edema) which are usually proportional in decompensation of chronic HF with low EF; however, up to 1 in 4 patients have a mismatch between right- and left-sided filling pressures. Hypoperfusion can be suspected from narrow pulse pressure and cool extremities, intolerance to neurohormonal antagonists, worsening renal function, altered mental status, and/or an elevated serum lactate. For more on the bedside evaluation of heart failure, enjoy Episode #142 – The Role of the Clinical Examination in Patients With Heart Failure – with Dr. Mark Drazner. Choice B, searching for precipitating factors is also correct. In patients hospitalized with HF, the common precipitating factors and the overall patient trajectory should be assessed to guide appropriate therapy (Class 1, LOE C-LD). Common precipitating factors include ischemic and nonischemic causes, such as acute coronary syndromes, atrial fibrillation and other arrhythmias, uncontrolled HTN, other cardiac disease (e.g., endocarditis), acute infections, anemia, thyroid dysfunction, non-adherence to medications or new medications. When initial clinical assessment does not suggest congestion or hypoperfusion, symptoms of HF may be a result of transient ischemia, arrhythmias, or noncardiac disease such as chronic pulmonary disease or pneumonia,
The following question refers to Section 9.5 of the 2022 AHA/ACC/HFSA Guideline for the Management of Heart Failure. The question is asked by Western Michigan University medical student & CardioNerds Intern Shivani Reddy, answered first by Brigham & Women's medicine resident and Director of CardioNerds Internship Dr. Gurleen Kaur, and then by expert faculty Dr. Shashank Sinha. Dr. Sinha is an Assistant Professor of Medical Education at the University of Virginia School of Medicine and an advanced heart failure, MCS, and transplant cardiologist at Inova Fairfax Medical Campus. He currently serves as both the Director of the Cardiac Intensive Care Unit and Cardiovascular Critical Care Research Program at Inova Fairfax. He is also a Steering Committee member for the multicenter Cardiogenic Shock Working Group and Critical Care Cardiology Trials Network and an Associate Editor for the Journal of Cardiac Failure, the official Journal of the Heart Failure Society of America. The Decipher the Guidelines: 2022 AHA / ACC / HFSA Guideline for The Management of Heart Failure series was developed by the CardioNerds and created in collaboration with the American Heart Association and the Heart Failure Society of America. It was created by 30 trainees spanning college through advanced fellowship under the leadership of CardioNerds Cofounders Dr. Amit Goyal and Dr. Dan Ambinder, with mentorship from Dr. Anu Lala, Dr. Robert Mentz, and Dr. Nancy Sweitzer. We thank Dr. Judy Bezanson and Dr. Elliott Antman for tremendous guidance. Enjoy this Circulation 2022 Paths to Discovery article to learn about the CardioNerds story, mission, and values. Question #12 Mr. Shock is a 65-year-old man with a history of hypertension and non-ischemic cardiomyopathy (LVEF 25%) who is admitted with acute decompensated heart failure. He is currently being diuresed with a bumetanide drip, but is only making 20 cc/hour of urine. On exam, blood pressure is 85/68 mmHg and heart rate is 110 bpm. His JVP is at 12 cm and extremities are cool with thready pulses. Bloodwork is notable for a lactate of 3.5 mmol/L and creatinine of 2.5 mg/dL (baseline Cr 1.2 mg/dL). What is the most appropriate next step? A Augment diuresis with metolazone B Start sodium nitroprusside C Start dobutamine D Start oral metoprolol E None of the above Answer #12 Explanation The correct answer is C – start dobutamine. In this scenario, the patient is in cardiogenic shock given hypotension and evidence of end-organ hypoperfusion on exam and labs. The patient's cool extremities, low urine output, elevated lactate, and elevated creatinine all point towards hypoperfusion. In patients with cardiogenic shock, intravenous inotropic support should be used to maintain systemic perfusion and preserve end-organ function (Class 1, LOE B-NR). Further, in patients with cardiogenic shock whose end-organ function cannot be maintained by pharmacologic means, temporary MCS is reasonable to support cardiac function (Class 2a, LOE B-NR). The SCAI Cardiogenic Shock Criteria can be used to divide patients into stages. Stage A is a patient at risk for cardiogenic shock but currently not with any signs or symptoms, for example, a patient presenting with a myocardial infarction without present evidence of shock. Stage B is “pre-shock” – this may be a patient who has volume overload, tachycardia, and hypotension but does not have hypoperfusion based on exam and lab evaluation. Stage C is classic cardiogenic shock – the cold and wet profile. Bedside findings for Stage C shock include cool extremities, weak pulses, altered mental status, decreased urine output, and/or respiratory distress. Lab findings include impaired renal function, increased lactate, increased hepatic enzymes, and/or acidosis. Stage D is deteriorating with worsening hypotension and hypoperfusion with escalating use of pressors or mechanical circulatory support.
The following question refers to Section 8.1 of the 2022 AHA/ACC/HFSA Guideline for the Management of Heart Failure. The question is asked by Western Michigan University medical student & CardioNerds Intern Shivani Reddy, answered first by Brigham & Women's medicine resident and Director of CardioNerds Internship Dr. Gurleen Kaur, and then by expert faculty Dr. Prateeti Khazanie. Dr. Khazanie is an Associate Professor and Advanced Heart Failure and Transplant Cardiologist at the University of Colorado. She was an undergraduate at Duke University as a B.N. Duke Scholar. She spent two years at the NIH in the lab of Dr. Anthony Fauci and completed a dual MD-MPH program at Duke Medical School. When she started residency, she thought she was going to be an ID doctor, but she fell in love with cardiology at Stanford where she was an intern, resident, and then chief resident. She went back to Duke for her general cardiology and advanced heart failure/transplant fellowships as well as research training at the DCRI. Dr. Khazanie joined the University of Colorado in 2015 as a health services clinician researcher with a focus on improving health equity and bioethics in advanced heart failure care. She mentors medical students, residents, and fellows and is a faculty mentor for the University of Colorado Cardiology Fellows “House of Cards” mentoring group. She has research funding from the NIH/NHLBI K23, NIH Ethics Grant, and Ludeman Center for Women's Health Research. Dr. Khazanie is an author on the 2022 ACC/AHA/HFSA HF Guidelines, the 2021 HFSA Universal Definition of Heart Failure, and multiple scientific statements. The Decipher the Guidelines: 2022 AHA / ACC / HFSA Guideline for The Management of Heart Failure series was developed by the CardioNerds and created in collaboration with the American Heart Association and the Heart Failure Society of America. It was created by 30 trainees spanning college through advanced fellowship under the leadership of CardioNerds Cofounders Dr. Amit Goyal and Dr. Dan Ambinder, with mentorship from Dr. Anu Lala, Dr. Robert Mentz, and Dr. Nancy Sweitzer. We thank Dr. Judy Bezanson and Dr. Elliott Antman for tremendous guidance. Enjoy this Circulation 2022 Paths to Discovery article to learn about the CardioNerds story, mission, and values. Question #11 A 64-year-old woman with a history of chronic systolic heart failure secondary to NICM (LVEF 15-20%) s/p dual chamber ICD presents for routine follow-up. She reports several months of progressive fatigue, dyspnea, and peripheral edema. She has been hospitalized twice in the past year with acute decompensated heart failure. Efforts to optimize guideline directed medical therapy have been tempered by episodes of lightheadedness and hypotension. Her exam is notable for an elevated JVP, an S3 heart sound, and a III/VI holosystolic murmur best heard at the apex with radiation to the axilla. Labs show Na 130 mmol/L, Cr 1.8 mg/dL (from 1.1 mg/dL 6 months prior), and NT-proBNP 1,200 pg/mL. ECG in clinic shows sinus rhythm and a nonspecific IVCD with QRS 116 ms. Her most recent TTE shows biventricular dilation with LVEF 15-20%, moderate functional MR, moderate functional TR and estimated RVSP of 40mmHg. What is the most appropriate next step in management? A Refer to electrophysiology for upgrade to CRT-D B Increase sacubitril-valsartan dose C Refer for advanced therapies evaluation D Start treatment with milrinone infusion Answer #11 Explanation The correct answer is C – refer for advanced therapies evaluation. Our patient has multiple signs and symptoms of advanced heart failure including NYHA Class III-IV functional status, persistently elevated natriuretic peptides, severely reduced LVEF, evidence of end organ dysfunction, multiple hospitalizations for ADHF, edema despite escalating doses of diuretics, and progressive intolerance to GDMT. Importantly, the 2018 European Society of Cardiology revised definition of advanced HF focuses...
The following question refers to Section 7.7 of the 2022 AHA/ACC/HFSA Guideline for the Management of Heart Failure. The question is asked by St. George's University medical student and CardioNerds Intern Chelsea Tweneboah, answered first by Baylor College of Medicine Cardiology Fellow and CardioNerds Ambassador Dr. Jamal Mahar, and then by expert faculty Dr. Michelle Kittleson. Dr. Kittleson is Director of Education in Heart Failure and Transplantation, Director of Heart Failure Research, and Professor of Medicine at the Smidt Heart Institute, Cedars-Sinai. She is Deputy Editor of the Journal of Heart and Lung Transplantation, on Guideline Writing Committees for the American College of Cardiology (ACC)/American Heart Association, is the Co Editor-in-Chief for the ACC Heart Failure Self-Assessment Program, and on the Board of Directors for the Heart Failure Society of America. Her Clinician's Guide to the 2022 Heart Failure guidelines, published in the Journal of Cardiac Failure, are a must-read for everyone! The Decipher the Guidelines: 2022 AHA / ACC / HFSA Guideline for The Management of Heart Failure series was developed by the CardioNerds and created in collaboration with the American Heart Association and the Heart Failure Society of America. It was created by 30 trainees spanning college through advanced fellowship under the leadership of CardioNerds Cofounders Dr. Amit Goyal and Dr. Dan Ambinder, with mentorship from Dr. Anu Lala, Dr. Robert Mentz, and Dr. Nancy Sweitzer. We thank Dr. Judy Bezanson and Dr. Elliott Antman for tremendous guidance. Enjoy this Circulation 2022 Paths to Discovery article to learn about the CardioNerds story, mission, and values. Question #10 Ms. Heffpefner is a 54-year-old woman who comes to your office for a routine visit. She does report increased fatigue and dyspnea on exertion without new orthopnea or extremity edema. She was previously diagnosed with type 2 diabetes, morbid obesity, obstructive sleep apnea, and TIA. She is currently prescribed metformin 1000mg twice daily, aspirin 81mg daily, rosuvastatin 40mg nightly, and furosemide 40mg daily. In clinic, her BP is 140/85 mmHg, HR is 110/min (rhythm irregularly irregular, found to be atrial fibrillation on ECG), and BMI is 43 kg/m2. Transthoracic echo shows an LVEF of 60%, moderate LV hypertrophy, moderate LA enlargement, and grade 2 diastolic dysfunction with no significant valvulopathy. What is the best next step? A Provide reassurance B Refer for gastric bypass C Refer for atrial fibrillation ablation D Start metoprolol and apixaban Answer #10 Explanation The correct answer is D – start metoprolol and apixaban. Ms. Hefpeffner has a new diagnosis of atrial fibrillation (AF) and has a significantly elevated risk for embolic stroke based on her CHA2DS2-VASc score of 6 (hypertension, diabetes, heart failure, prior TIA, and female sex). The relationship between AF and HF is complex and they the presence of either worsens the status of the other. Managing AF in patients with HFpEF can lead to symptom improvement (Class 2a, LOR C-EO). However, large, randomized trial data are unavailable to specifically guide therapy in patients with AF and HFpEF. Generally, management of AF involves stroke prevention, rate and/or rhythm control, and lifestyle / risk-factor modification. With regards to stroke prevention, patients with chronic HF with permanent-persistent-paroxysmal AF and a CHA2DS2-VASc score of ≥2 (for men) and ≥3 (for women) should receive chronic anticoagulant therapy (Class 1, LOE A). When anticoagulation is used in chronic HF patients with AF, DOAC is recommended over warfarin in eligible patients (Class 1, LOE A). The decision for rate versus rhythm control should be individualized and reflects both patient symptoms and the likelihood of better ventricular function with sinus rhythm. For patients with HF and symptoms caused by AF, AF ablation is reasonable to improve symptoms and QOL (Class 2a,
The following question refers to Section 7.6 of the 2022 AHA/ACC/HFSA Guideline for the Management of Heart Failure. The question is asked by premedical student and CardioNerds Intern Pacey Wetstein, answered first by Baylor College of Medicine Cardiology Fellow and CardioNerds Ambassador Dr. Jamal Mahar, and then by expert faculty Dr. Nancy Sweitzer. Dr. Sweitzer is Professor of Medicine, Vice Chair of Clinical Research for the Department of Medicine, and Director of Clinical Research for the Division of Cardiology at Washington University School of Medicine. She is the editor-in-chief of Circulation: Heart Failure. Dr. Sweitzer is a faculty mentor for this Decipher the HF Guidelines series. The Decipher the Guidelines: 2022 AHA / ACC / HFSA Guideline for The Management of Heart Failure series was developed by the CardioNerds and created in collaboration with the American Heart Association and the Heart Failure Society of America. It was created by 30 trainees spanning college through advanced fellowship under the leadership of CardioNerds Cofounders Dr. Amit Goyal and Dr. Dan Ambinder, with mentorship from Dr. Anu Lala, Dr. Robert Mentz, and Dr. Nancy Sweitzer. We thank Dr. Judy Bezanson and Dr. Elliott Antman for tremendous guidance. Enjoy this Circulation 2022 Paths to Discovery article to learn about the CardioNerds story, mission, and values. Question #9 Mr. Flo Zin is a 64-year-old man who comes to discuss persistent lower extremity edema and dyspnea with mild exertion. He takes amlodipine for hypertension but has no other known comorbidities. In the clinic, his heart rate is 52 bpm and blood pressure is 120/70 mmHg. Physical exam reveals mildly elevated jugular venous pulsations and 1+ bilateral lower extremity edema. Labs show an unremarkable CBC, normal renal function and electrolytes, a Hb A1c of 6.1%, and an NT-proBNP of 750 (no prior baseline available). On echocardiogram, his LVEF is 44% and nuclear stress testing was negative for inducible ischemia. What is the best next step in management? A Add furosemide BID and daily metolazone B Start empagliflozin and furosemide as needed C Start metoprolol succinate D No change to medical therapy Answer #9 Explanation The correct answer is B – start empagliflozin and furosemide as needed. The patient described here has heart failure with mildly reduced EF (HFmrEF), given LVEF in the range of 41-49%. In patients with HF who have fluid retention, diuretics are recommended to relieve congestion, improve symptoms, and prevent worsening HF (Class 1, LOE B-NR). For patients with HF and congestive symptoms, addition of a thiazide (eg, metolazone) to treatment with a loop diuretic should be reserved for patients who do not respond to moderate or high-dose loop diuretics to minimize electrolyte abnormalities (Class 1, LOE B-NR). Therefore, option A is not correct as he is only mildly congested on examination, and likely would not require such aggressive decongestive therapy, particularly with normal renal function. Adding a thiazide diuretic without first optimizing loop diuretic dosing would be premature. The EMPEROR-Preserved trial showed a significant benefit of the SGLT2i, empagliflozin, in patients with symptomatic HF, with LVEF >40% and elevated natriuretic peptides. The 21% reduction in the primary composite endpoint of time to HF hospitalization or cardiovascular death was driven mostly by a significant 29% reduction in time to HF hospitalization, with no benefit on all-cause mortality. Empagliflozin also resulted in a significant reduction in total HF hospitalizations, decrease in the slope of the eGFR decline, and a modest improvement in QOL at 52 weeks. Of note, the benefit was similar irrespective of the presence or absence of diabetes at baseline. In a subgroup of 1983 patients with LVEF 41% to 49% in EMPEROR-Preserved, empagliflozin, an SGLT2i, reduced the risk of the primary composite endpoint of cardiovascular death or hospitalization f...
The following question refers to Section 7.4 of the 2022 AHA/ACC/HFSA Guideline for the Management of Heart Failure. The question is asked by New York Medical College medical student and CardioNerds Intern Akiva Rosenzveig, answered first by Cornell cardiology fellow and CardioNerds Ambassador Dr. Jaya Kanduri, and then by expert faculty Dr. Randall Starling.Dr. Starling is Professor of Medicine and an advanced heart failure and transplant cardiologist at the Cleveland Clinic where he was formerly the Section Head of Heart Failure, Vice Chairman of Cardiovascular Medicine, and member of the Cleveland Clinic Board of Governors. Dr. Starling is also Past President of the Heart Failure Society of America in 2018-2019. Dr. Staring was among the earliest CardioNerds faculty guests and has since been a valuable source of mentorship and inspiration. Dr. Starling's sponsorship and support was instrumental in the origins of the CardioNerds Clinical Trials Program.The Decipher the Guidelines: 2022 AHA / ACC / HFSA Guideline for The Management of Heart Failure series was developed by the CardioNerds and created in collaboration with the American Heart Association and the Heart Failure Society of America. It was created by 30 trainees spanning college through advanced fellowship under the leadership of CardioNerds Cofounders Dr. Amit Goyal and Dr. Dan Ambinder, with mentorship from Dr. Anu Lala, Dr. Robert Mentz, and Dr. Nancy Sweitzer. We thank Dr. Judy Bezanson and Dr. Elliott Antman for tremendous guidance.Enjoy this Circulation 2022 Paths to Discovery article to learn about the CardioNerds story, mission, and values. Question #6 Mr. D is a 50-year-old man who presented two months ago with palpations and new onset bilateral lower extremity swelling. Review of systems was negative for prior syncope. On transthoracic echocardiogram, he had an LVEF of 40% with moderate RV dilation and dysfunction. EKG showed inverted T-waves and low-amplitude signals just after the QRS in leads V1-V3. Ambulatory monitor revealed several episodes non-sustained ventricular tachycardia with a LBBB morphology. He was initiated on GDMT and underwent genetic testing that revealed 2 desmosomal gene variants associated with arrhythmogenic right ventricular cardiomyopathy (ARVC). Is the following statement true or false? “ICD implantation is inappropriate at this time because his LVEF is >35%” True False Answer #6 Explanation This statement is False. ICD implantation is reasonable to decrease sudden death in patients with genetic arrhythmogenic cardiomyopathy with high-risk features of sudden death who have an LVEF ≤45% (Class 2a, LOE B-NR). While the HF guidelines do not define high-risk features of sudden death, the 2019 HRS expert consensus statement on evaluation, risk stratification, and management of arrhythmogenic cardiomyopathy identify major and minor risk factors for ventricular arrhythmias as follows: Major criteria: NSVT, inducibility of VT during EPS, LVEF ≤ 49%. Minor criteria: male sex, >1000 premature ventricular contractions (PVCs)/24 hours, RV dysfunction, proband status, 2 or more desmosomal variants. According to the HRS statement, high risk is defined as having either three major, two major and two minor, or one major and four minor risk factors for a class 2a recommendation for primary prevention ICD in this population (LOE B-NR). Based on these criteria, our patient has 2 major risk factors (NSVT & LVEF ≤ 49%), and 3 minor risk factors (male sex, RV dysfunction, and 2 desmosomal variants) for ventricular arrhythmias. Therefore, ICD implantation for primary prevention of sudden cardiac death is reasonable. Decisions around ICD implantation for primary prevention remain challenging and depend on estimated risk for SCD, co-morbidities, and patient preferences, and so should be guided by shared decision making weighing the possible benefits against the risks,
The following question refers to Section 7.1 of the 2022 AHA/ACC/HFSA Guideline for the Management of Heart Failure. The question is asked by New York Medical College medical student and CardioNerds Intern Akiva Rosenzveig, answered first by Cornell cardiology fellow and CardioNerds Ambassador Dr. Jaya Kanduri, and then by expert faculty Dr. Clyde Yancy.Dr. Yancy is Professor of Medicine and Medical Social Sciences, Chief of Cardiology, and Vice Dean for Diversity and Inclusion at Northwestern University, and a member of the AHA/ACC/HFSA Heart Failure Guideline Writing Committee.The Decipher the Guidelines: 2022 AHA / ACC / HFSA Guideline for The Management of Heart Failure series was developed by the CardioNerds and created in collaboration with the American Heart Association and the Heart Failure Society of America. It was created by 30 trainees spanning college through advanced fellowship under the leadership of CardioNerds Cofounders Dr. Amit Goyal and Dr. Dan Ambinder, with mentorship from Dr. Anu Lala, Dr. Robert Mentz, and Dr. Nancy Sweitzer. We thank Dr. Judy Bezanson and Dr. Elliott Antman for tremendous guidance.Enjoy this Circulation 2022 Paths to Discovery article to learn about the CardioNerds story, mission, and values. Question #5 Ms. L is a 65-year-old woman with nonischemic cardiomyopathy with a left ventricular ejection fraction (LVEF) of 35%, hypertension, and type 2 diabetes mellitus. She has been admitted to the hospital with decompensated heart failure (HF) twice in the last six months and admits that she struggles to understand how to take her medications and adjust her sodium intake to prevent this. Which of the following interventions has the potential to decrease the risk of rehospitalization and/or improve mortality? A Access to a multidisciplinary team (physicians, nurses, pharmacists, social workers, care managers, etc) to assist with management of her HF B Engaging in a mobile app aimed at improving HF self-care C Vaccination against respiratory illnesses D A & C Answer #5 The correct answer is D – both A (access to a multidisciplinary team) and C (vaccination against respiratory illness). Choice A is correct. Multidisciplinary teams involving physicians, nurses, pharmacists, social workers, care managers, dieticians, and others, have been shown in multiple RCTs, metanalyses, and Cochrane reviews to both reduce hospital admissions and all-cause mortality. As such, it is a class I recommendation (LOE A) that patients with HF should receive care from multidisciplinary teams to facilitate the implementation of GDMT, address potential barriers to self-care, reduce the risk of subsequent rehospitalization for HF, and improve survival. Choice B is incorrect. Self-care in HF comprises treatment adherence and health maintenance behaviors. Patients with HF should learn to take medications as prescribed, restrict sodium intake, stay physically active, and get vaccinations. They also should understand how to monitor for signs and symptoms of worsening HF, and what to do in response to symptoms when they occur. Interventions focused on improving the self-care of HF patients significantly reduce hospitalizations and all-cause mortality as well as improve quality of life. Therefore, patients with HF should receive specific education and support to facilitate HF self-care in a multidisciplinary manner (Class I, LOE B-R). However, the method of delivery and education matters. Reinforcement with structured telephone support has been shown to be effective. In contrast the efficacy of mobile health-delivered educational interventions in improve self-care in patients with HF remains uncertain. Choice C is correct. In patients with HF, vaccinating against respiratory illnesses is reasonable to reduce mortality (Class 2a, LOE B-NR). For example, administration of the influenza vaccine in HF patients has been shown to reduce...
The following question refers to Section 6.1 of the 2022 AHA/ACC/HFSA Guideline for the Management of Heart Failure. The question is asked by Keck School of Medicine USC medical student & CardioNerds Intern Hirsh Elhence, answered first by Mount Sinai Hospital cardiology fellow and CardioNerds FIT Trialist Dr. Jason Feinman, and then by expert faculty Dr. Mark Drazner. Dr. Drazner is an advanced heart failure and transplant cardiologist, Professor of Medicine, and Clinical Chief of Cardiology at UT Southwestern. He is the President of the Heart Failure Society of America. The Decipher the Guidelines: 2022 AHA / ACC / HFSA Guideline for The Management of Heart Failure series was developed by the CardioNerds and created in collaboration with the American Heart Association and the Heart Failure Society of America. It was created by 30 trainees spanning college through advanced fellowship under the leadership of CardioNerds Cofounders Dr. Amit Goyal and Dr. Dan Ambinder, with mentorship from Dr. Anu Lala, Dr. Robert Mentz, and Dr. Nancy Sweitzer. We thank Dr. Judy Bezanson and Dr. Elliott Antman for tremendous guidance. Enjoy this Circulation 2022 Paths to Discovery article to learn about the CardioNerds story, mission, and values. Question #2 A 67-year-old man with a past medical history of type 2 diabetes mellitus, hypertension, and active tobacco smoking presents to the emergency room with substernal chest pain for the past 5 hours. An electrocardiogram reveals ST segment elevations in the anterior precordial leads and he is transferred emergently to the catheterization laboratory. Coronary angiography reveals 100% occlusion of the proximal left anterior descending artery, and he is successfully treated with a drug eluting stent resulting in TIMI 3 coronary flow. Following his procedure, a transthoracic echocardiogram is performed which reveals a left ventricular ejection fraction of 35% with a hypokinetic anterior wall. Which of the following medications would be the best choice to prevent the incidence of heart failure and reduce mortality? A Lisinopril B Diltiazem C Carvedilol D Sacubitril-valsartan E Both A and C Answer #2 The correct answer is E – both lisinopril and carvedilol are appropriate to reduce the incidence of heart failure and mortality. Evidence-based beta-blockers and ACE inhibitors both have Class 1 recommendations in patients with a recent myocardial infarction and left ventricular ejection fraction ≤ 40% to reduce the incidence of heart failure and to reduce mortality. Multiple randomized controlled trials have investigated both medications in the post myocardial infarction setting and demonstrated improved ventricular remodeling as well as benefits for mortality and development of incident heart failure. At this time, there is not sufficient evidence to recommend ARNi over ACEi for patients with reduced LVEF following acute MI. The PARADISE-MI trial randomized a total of 5,661 patients with myocardial infarction complicated by a reduced LVEF, pulmonary congestion, or both to receive either sacubitril-valsartan (97-103mg twice daily) or ramipril (5mg twice daily). After a median follow up time of 22 months, there was no statistically significant difference in the primary outcome of cardiovascular death or incident heart failure. At this time, ARNi have not been included in the guidelines for this specific population. Diltiazem is a non-dihydropyridine calcium channel blocker, a family of drugs with negative inotropic effects and which may be harmful in patients with depressed LVEF (Class 3: Harm, LOE C-LD). Main Takeaway: For patients with recent myocardial infarction and reduced left ventricular function both beta blockers and ACEi have Class 1 recommendations to reduce the incidence of heart failure and decrease mortality. Guideline Location: Section 6.1